Neurology Flashcards

1
Q
Epidural hematoma is caused by the rupture of:
A)  	middle meningeal artery
B)  	choroidal anterior artery
C)  	middle cerebral artery
D)  	superficial temporal artery
A

A) middle meningeal artery

EXPLANATION
The middle menigeal artery is the branch of external carotid artery and runs between the dura mater and the temporal bone. In case of bone fracture (temporal or parietal region), the middle meningeal artery could be ruptured and arterial hematoma develops quickly by compressing the ipsilateral hemisphere resulting in first contralateral hemiparesis thereafter somnolence, stupor, coma and finally herniation.

How well did you know this?
1
Not at all
2
3
4
5
Perfectly
2
Q

The importance of external carotid artery is:
A) important for the blood flow supply of posterior scala
B) important potential collateral source in case of ipsilateral ICA occlusion
C) participates in blood supply of brainstem
D) its occlusion results in amaurosis fugax
E) supplies the frontobasal part of the brain

A

B) important potential collateral source in case of ipsilateral ICA occlusion

EXPLANATION
The collateral circulation between the branches of external carotid artery (facial, angular arteries etc.) and ophthalmic artery (ICA branch) is important. In case of severe ICA stenosis or occlusion the good collateral circulation can sustain asymptomatic status. The reversed flow (extra-intracranial direction) in the ophthalmic artery can be detected by ultrasound.

How well did you know this?
1
Not at all
2
3
4
5
Perfectly
3
Q
The risk of stroke in hypertension:
A)  	2–8x
B)  	0,3–3x
C)  	2x
D)  	50x
A

A) 2–8x

EXPLANATIONThe hypertension increases the stroke risk by 2-8x

How well did you know this?
1
Not at all
2
3
4
5
Perfectly
4
Q
Symptoms of TIA, EXCEPT:
A)  	Transient unilateral blindness
B)  	Transient limb numbness
C)  	Transient aphasia
D)  	Transient loss of consciousness with epileptic seizure
A

D) Transient loss of consciousness with epileptic seizure

EXPLANATION
Transient focal deficits (unilateral blindness, aphasia, transient focal numbness, transient paresis) are typical symptoms of TIA. A transient loss of consciousness could be caused by quick blood pressure decrease, arhythmias (pump-function), blood glucose decrease or increase, and epilepsy but not by TIA.

How well did you know this?
1
Not at all
2
3
4
5
Perfectly
5
Q
Cerebral blood flow in the penumbra (region around the ischemic core)
A)  	0–10 ml/min/100 g brain tissue
B)  	10–20 ml/min/100 g brain tissue
C)  	55–60 ml/min/100 g brain tissue
D)  	100-120/min/100 g brain tissue
A

B) 10–20 ml/min/100 g brain tissue

EXPLANATION
The iv. thrombolysis or thrombectomy target the penumbra (10-20 ml/min/100g brain tissue, in healthy person 50ml/100g/min). The structures of the neurons are still preserved with impaired function.

How well did you know this?
1
Not at all
2
3
4
5
Perfectly
6
Q
Global cerebral ischemia causes irreversible cerebral damage after:
A)  	20–25 min
B)  	10–15 min
C)  	3–5 min
D)  	1–2 min
A

ANSWER
C) 3–5 min

EXPLANATION
Globalis cerebral ischemia causes irreversible cerebral damage after 3-5 minutes

How well did you know this?
1
Not at all
2
3
4
5
Perfectly
7
Q
Which mechanism plays an important role in the ischemic cascade?
A)  	calcium influx
B)  	potassium influx
C)  	sodium influx
D)  	decrease of monoamine level
A

A) calcium influx
EXPLANATION
During acute ischemia the calcium influx activates the ischemic cascade.

How well did you know this?
1
Not at all
2
3
4
5
Perfectly
8
Q

The carotid ultrasound is recommended in acute stroke, EXCEPT:
A) for the measurement of intima-media thickness.
B) to diagnose carotid occlusion/stenosis.
C) to determine the plaque characteristic: in case of ulcerated or inhomogenous plaque, endareterectomy could be suggested instead of stenting.
D) because it is the optimal method to detect carotid floating thrombus.

A

A) for the measurement of intima-media thickness.

EXPLANATION
The intima-media thickness is a marker of arteriosclerosis but not the part of acute stroke care.

How well did you know this?
1
Not at all
2
3
4
5
Perfectly
9
Q
Symptoms of vertebrobasilar insufficiency EXCEPT.
A)  	diplopia
B)  	vertigo
C)  	dysarthria
D)  	alternating brainstem syndromes
E)  	apraxia
A

E) apraxia

EXPLANATION
A supratentorial lesion results in apraxia but not infratentorial one.

How well did you know this?
1
Not at all
2
3
4
5
Perfectly
10
Q

Typical symptoms of transient global amnesia, EXCEPT:

A)  	loss of anterograde memory
B)  	the patient is alert
C)  	shorter than 24 hours
D)  	cortical blindness
E)  	unknown cause
A

D) cortical blindness

EXPLANATION
Cortical blindness is caused by the bilateral occlusion of the posterior cerebral artery, which is not typical for transient global amnesia (TGA).

How well did you know this?
1
Not at all
2
3
4
5
Perfectly
11
Q

Cause of amaurosis fugax:
A) giant cell arteriitis
B) migraine with scotoma
C) thromboembolisation of the ophthalmic artery (terminal branch) from the ulcerated plaque of the ipsilateral internal carotid artery

A

C) thromboembolisation of the ophthalmic artery (terminal branch) from the ulcerated plaque of the ipsilateral internal carotid artery

EXPLANATION
The thromboembolisation of the ophthalmic artery may origin from the ulcerated plaque of the internal carotid artery, causing ipsilateral amaurosis fugax (transient monocular blindness). Contrarily occlusion of the calcarina artery causes contralateral heteronymous hemianopia, in case of migraine scotoma can develop. In case of giant cell arteriitis the lesion of the anterior part of the optic nerve or lesion of the central retinal artery is responsible for the deterioration of visual acuity.

How well did you know this?
1
Not at all
2
3
4
5
Perfectly
12
Q

Cortical lesion is probable in case of stroke in the territory of the internal carotid artery if:
A) hemiparesis with dominance in the facial and brachial area
B) if the severity of the paresis is similar on the upper and lower limb
C) visual field defect is also seen
D) permanent vertigo develops

A

A) hemiparesis with dominance in the facial and brachial area

EXPLANATION
If facial-brachial-dominant hemiparesis is seen in case of stroke affecting the territory of the internal carotid artery cortical lesion is probable, whilst the severity is similar in the face, upper limb and lower limb subcortical lesion (internal capsule) is more likely.

How well did you know this?
1
Not at all
2
3
4
5
Perfectly
13
Q

Which is NOT part of the symptoms of the unilateral occlusion of the anterior cerebral artery and its branches?
A) urinary and fecal incontinence
B) contralateral lower limb dominant hemiparesis
C) changes in behaviour and character
D) gnostic disturbance

A

D) gnostic disturbance

EXPLANATION
Gnostic dysfunction appear in lesions next to the primary sensory centers, supplied by the branches of the middle cerebral artery.

How well did you know this?
1
Not at all
2
3
4
5
Perfectly
14
Q
Which is NOT part of the symptoms in case of occlusion of the middle cerebral artery (M1)?
A)  	contralateral severe hemiparesis
B)  	homonymous hemianopia
C)  	conjugate deviation of eyes
D)  	hemihypaesthesia
E)  	thalamus-syndrome, thalamus-hand
A

E) thalamus-syndrome, thalamus-hand

EXPLANATION
Thalamus-syndrome, thalamus-hand occur as a consequence of the occlusion of the branches of the thalamogeniculate artery (branches of the posterior cerebral artery), when the ventral posteromedial and posterolateral thalamic nuclei are damaged.

How well did you know this?
1
Not at all
2
3
4
5
Perfectly
15
Q
NOT part of Weber-syndrome:
A)  	ipsilateral oculomotor nerve lesion
B)  	contralateral hemiparesis
C)  	contralateral increase deep tendon reflexes with pyramidal signs
D)  	contralateral hyperkinesia
A

D) contralateral hyperkinesia

EXPLANATION
Weber-syndrome is an alternating motor brainstem syndrome: ipsilateral oculomotor nerve lesion, contralateral hemiparesis. When contralateral involuntary movements (hyperkinesis) occur not only the oculomotor nerve, but the red nucleus is damaged as well (Benedikt-syndrome).

How well did you know this?
1
Not at all
2
3
4
5
Perfectly
16
Q
Disturbance of gaze (vertical upwards), with vertical nystagmus is typical:
A)  	Benedikt-syndrome
B)  	Parinaud-syndrome
C)  	Nothnagel-syndrome
D)  	Raymond- (ventral pontine) syndrome
A

B) Parinaud-syndrome

EXPLANATION
The disturbance of gaze (vertical upwards) with vertical nystagmus is typical for Parinaud-syndrome. In the background tumor (especially pinealoma), sometimes vascular lesion and inflammation is found.

How well did you know this?
1
Not at all
2
3
4
5
Perfectly
17
Q

Cause of pseudobulbar palsy:
A) bilateral lesion of the corticobulbar fibers
B) lesion of the cranial nerves in the medulla oblongata
C) lesion of the dentate nucleus
D) lesion of the peripheral nerves responsible for articulation (speech) and swallowing

A

A) bilateral lesion of the corticobulbar fibers

EXPLANATIONBilateral lesion of the corticobulbar tracts causes pseudobulbar palsy.

How well did you know this?
1
Not at all
2
3
4
5
Perfectly
18
Q

Symptoms of occlusion of the superior cerebellar artery (lateral superior pontine syndrome), EXCEPT:
A) ipsilateral limb and truncal ataxia
B) dizziness, nystagmus
C) contralateral hypaesthesia, decreased vibration and joint position sensation
D) diplopia, deafness

A

D) diplopia, deafness
EXPLANATION
The following symptoms are detected in case of occlusion of the cerebellar superior artery (lateral superior pontine syndrome): Ipsilateral limb- and truncal ataxia, vertigo, horizontal nystagmus, contralateral hypaesthesia, vibration, joint position sensation The lesion may affect the superior and middle cerebellar peduncles, dentate nucleus, the vestibular nuclei of the cerebellum, the spinothalamic tract, dorsal part of medial lemniscus and the descending sympathic fibers.

How well did you know this?
1
Not at all
2
3
4
5
Perfectly
19
Q

What does Hunt and Hess Scale grade 4 stand for?

A) somnolence, confusion, mild neurological symptoms
B) deep coma, decerebration
C) stupor, moderate or severe hemiparesis, vegetative disturbance
D) severe headache, nuchal rigidity, cranial nerve palsy

A

ANSWER
C) stupor, moderate or severe hemiparesis, vegetative disturbance
EXPLANATION
By Hunt and Hess Scale grade 4 stupor, moderate or severe hemiparesis, vegetative disturbance are the symptoms. It grades the severity of subarachnoid hemorrhage.

How well did you know this?
1
Not at all
2
3
4
5
Perfectly
20
Q
Which artery’s aneurysm may cause unilateral oculomotor nerve palsy?
A)  	posterior communicating artery
B)  	anterior communicating artery
C)  	ophthalmic artery
D)  	posterior cerebral artery
A

A) posterior communicating artery

EXPLANATION
Unilateral total oculomotor nerve lesion can be caused by the aneurysm of the posterior communicating artery, because of its localization it may compress the oculomotor nerve.

How well did you know this?
1
Not at all
2
3
4
5
Perfectly
21
Q
What type of electrolyte abnormality may develop after subarachnoid haemorrhage?
A)  	hypokalaemia
B)  	hyponatraemia
C)  	both of them
D)  	none of them
A

B) hyponatraemia
EXPLANATION
Often hyponatraemia develops after subarachnoid haemorrhage (because of the disturbance of ADH-secretion).

How well did you know this?
1
Not at all
2
3
4
5
Perfectly
22
Q
Where the bleeding is localized in case of the following symptoms: eye-balls in the midline, bilateral miosis and pupils react poorly to light.
A)  	thalamus
B)  	pons
C)  	cerebellum
D)  	putamen
A

B) pons
EXPLANATION
In case of intracranial haemorrhage, if the bulbi are in central position, the pupils are point like (myosis), and show poor reaction to light, the bleeding is in the pons probably.

How well did you know this?
1
Not at all
2
3
4
5
Perfectly
23
Q
On cranial CT in the so called watershed area hypodens lesion is detected, with hyperdense petechia. This is typical for:
A)  	bleeding
B)  	chronic infarct
C)  	fresh infarct
D)  	haemorrhagic infarct
E)  	cavernoma
A

D) haemorrhagic infarct

EXPLANATION
Haemorrhagic infarct is the diagnosis if on cranial CT in the so called watershed zone hypodensity is seen with smaller hyperdense territories inside.

How well did you know this?
1
Not at all
2
3
4
5
Perfectly
24
Q
The following symptoms are typical for the occlusion of the ………. artery: short after backpain (but not ictally) flaccid paraplegia with dissociated disturbance of sensation (deep sensation is preserved), urinary and bowel incontinence.
A)  	posterior spinal artery
B)  	sulcocommissural artery
C)  	anterior spinal artery
D)  	none of them
A

C) anterior spinal artery

EXPLANATION
The typical symptoms of occlusion of anterior spinal artery are: short (but not ictal) after pain in the thoracic/lumbar region flaccid paraplegia develops, with dissociated sensation disturbance (deep sensation is intact), incontinence (fecal and urinary). Prodromal symptoms (pain in the altitude of the lesion) may be reported before the severe symptoms. Pyramidal signs may be missing at the beginning. A space occupying procedure may compress the anterior spinal artery e.g. disc herniation.

How well did you know this?
1
Not at all
2
3
4
5
Perfectly
25
Q
On auscultation of the carotid arteries no bruit can be heard in case of:
A)  	aortastenosis
B)  	70% stenosis of the carotid artery
C)  	occlusion of the carotid artery
D)  	inbleeded atherosclerotic plaque
A

C) occlusion of the carotid artery

EXPLANATION
Examining the supraaortic arteries by auscultation is mandatory screening method. A bruit draws attention to stenosis of the artery if it is not conducted from the heart or the aortic valve. Mild stenosis usually does not cause bruit, but the bruit may also cease when the stenosis is severe. subtotal or occluded. When the character of the bruit changes inbleeding plaque or dissection of the wall of a blood vessel may be in the background. All the above mentioned options need urgent examination (e.g. angiography).

How well did you know this?
1
Not at all
2
3
4
5
Perfectly
26
Q

By evaluating the severity of stenosis with duplex scan the measurement of velocity of the systolic and diastolic blood flow is necessary. When should one suspect more than 90% stenosis of the internal carotid artery?
A) if the systolic peak velocity is 110–120 cm/s
B) if the diastolic peak velocity is less than 40 cm/s
C) if the diastolic peak velocity is more than 100 cm/s
D) if the systolic velocity is less than110, diastolic less than 40 cm/s

A

C) if the diastolic peak velocity is more than 100 cm/s

EXPLANATION
By judging the exact size of the stenosis with duplex scan measuring the systolic and diastolic velocity of the flow is important. Above the systolic peak velocity is 110-120 cm/s, a diastolic peak velocity 40 cm/s stenosis is diagnosed. Above approximately 250 cm/s systolic and 100 cm/s diastolic velocity means a stenosis of more than 90%.

How well did you know this?
1
Not at all
2
3
4
5
Perfectly
27
Q

Alternating sensory /motor syndrome is typical of:
A) brainstem lesion
B) bilateral internal carotid artery occlusion
C) occlusion of the posterior cerebral artery
D) occlusion of the posterior choroidal artery

A

A) brainstem lesion

EXPLANATION
Appearance of alternating (crossed) sensory and/or motor symptoms show a localisation in the brainstem. The locus of the lesion is given by the cranial nerve’s symptom the contralateral „long tract” symptom is caused by the lesion of the tracts running in the base of the brainstem.

How well did you know this?
1
Not at all
2
3
4
5
Perfectly
28
Q

Lowering of elevated blood pressure in acute ischemic stroke is not recommended, except:
A) if the diastolic blood pressure exceeds 120 mmHg
B) if systolic blood pressure is 180 mmHg
C) if the symptoms improve dramatically
D) if the ultrasound test does not reveal significant internal carotid artery stenosis

A

A) if the diastolic blood pressure exceeds 120 mmHg

EXPLANATION
Lowering of elevated blood pressure is not recommended in acute ischemic stroke only if systolic blood pressure exceeds 190-200 mmHg and diastolic blood pressure exceeds 120 mmHg, because decreased cerebral perfusion pressure may cause adverse effects. However, after acute phase of ischemic stroke, hypertension must be treated. Often, high blood pressure decreases spontaneously in a few days.

How well did you know this?
1
Not at all
2
3
4
5
Perfectly
29
Q
From the onset of ischemic stroke symptoms, systemic thrombolysis can be indicated:
A)  	within 6 hours
B)  	within 12 hours
C)  	within 24 hours
D)  	within 4.5 hours
A

D) within 4.5 hours

EXPLANATION
From the development of ischemic stroke symptoms, thrombolysis may be indicated within 4.5 hours if the patient does not have any other exclusion criteria. If the symptoms are noted upon morning awakening, the onset time of stroke symptoms is uncertain. In this case, the last time point, when the patient was symptom-free, should be considered (e.g. bedtime, night awakening).

How well did you know this?
1
Not at all
2
3
4
5
Perfectly
30
Q

In acute ischemic stroke, based on various clinical studies, the following systemic thrombolytic agent has the best effect, with minimal bleeding complications:
A) urokinase
B) rt-PA (recombinant tissue - plasminogen activator)
C) streptokinase
D) ancrod

A

B) rt-PA (recombinant tissue - plasminogen activator)

EXPLANATION
In acute ischemic stroke, based on various clinical studies, systemic thrombolysis with rt-PA (recombinant tissue plasminogen activator) is the most effective method with the least bleeding complications.

How well did you know this?
1
Not at all
2
3
4
5
Perfectly
31
Q
Treatment of cerebral edema in ischemic stroke:
A)  	high-dose steroid
B)  	furosemid
C)  	mannitol
D)  	dextran
A

C) mannitol

EXPLANATION
Mannitol infusion (20%, 200 mg/kg body weight in 10-15 minutes) can be used to treat cerebral edema caused by brain ischemia. In severe cases, Mannitol administration can be repeated every 4-6 hours. After stop of the dehydration, a rebound effect might develop. To avoid renal failure no greater dose than 100 g of mannitol per day sholud be used.
How well did you know this?
1
Not at all
2
3
4
5
Perfectly
32
Q

How can the functional status in patient with cerebrovascular disease be evaluated?
A) with Mathew Scale
B) with Barthel index
C) with Canadian Stroke Scale
D) with Orgogozo Scale
E) with combined use of Unified and Motor Score Scales

A

B) with Barthel index

EXPLANATIONThe change of the functional status in patient with cerebrovascular disease can be evaluated by the Barthel scale.

How well did you know this?
1
Not at all
2
3
4
5
Perfectly
33
Q
What are the non-modifiable risk factors for stroke?
1)  	stress
2)  	TIA
3)  	alcoholism
4)  	left ventricular hypertrophy
A)  	1st, 2nd and 3rd answers are correct
B)  	1st and 3rd answers are correct
C)  	2nd and 4th answers are correct
D)  	only 4th answer is correct
E)  	all of the answers are correct
A

C) 2nd and 4th answers are correct

EXPLANATION
Non-modifiable risk factors for stroke include (but not limited to): TIA, left ventricular hypertrophy, previous myocardial infarction, peripheral vascular disease, age, gender.

How well did you know this?
1
Not at all
2
3
4
5
Perfectly
34
Q

The diaschisis could occur:

1) in the contralateral hemisphere
2) in the ipsilateral putamen, in case of frontal lesion
3) in the contralateral cerebellum
4) in the brainstem

A)  	1st, 2nd and 3rd answers are correct
B)  	1st and 3rd answers are correct
C)  	2nd and 4th answers are correct
D)  	only 4th answer is correct
E)  	all of the answers are correct
A

B) 1st and 3rd answers are correct

EXPLANATIONIn hemispheric damage diaschisis (loss of function in regions of the brain being in connection with a distant, impaired brain area) may occur in the contralateral brain hemisphere and in the contralateral cerebellar hemisphere.

How well did you know this?
1
Not at all
2
3
4
5
Perfectly
35
Q

Consequence(s) of subarachnoid haemorrhage:

1) vasospasm, leading to secondary ischemic damage
2) diaschisis effect
3) nonresorptive hydrocephalus
4) chronic subdural bleeding

A)  	1st, 2nd and 3rd answers are correct
B)  	1st and 3rd answers are correct
C)  	2nd and 4th answers are correct
D)  	only 4th answer is correct
E)  	all of the answers are correct
A

B) 1st and 3rd answers are correct

EXPLANATION
As a consequence of subarachnoid haemorrhage, vasospasm may develop in 15-45% of patients due to release of thromboxane, free radicals, serotonin etc. from the blood. Vasospasm usually appears 4-12 days after bleeding, and may lead to secondary ischemic damage. Other consequence of subarachnoid bleeding is inhibition of CSF circulation resulting in non-resorptive hydrocephalus.

How well did you know this?
1
Not at all
2
3
4
5
Perfectly
36
Q

Characteristics of Transient Ischemic Attack (TIA):

1) 10% of cerebrovascular diseases
2) in half the cases, the symptoms last for less than 30 minutes
3) in these patients the rate of myocardial infarction is high
4) with CT scan, lacunar infarction rate is above 50%

A)  	1st, 2nd and 3rd answers are correct
B)  	1st and 3rd answers are correct
C)  	2nd and 4th answers are correct
D)  	only 4th answer is correct
E)  	all of the answers are correct
A

A) 1st, 2nd and 3rd answers are correct

EXPLANATIONApproximately 10% of all cerebrovascular diseases is transient ischemic attack (TIA). Symptoms develop very rapidly, but rarely last longer than 1 hour, and in half the cases last less than 30 minutes. There is a high rate of myocardial infarction among patients with TIA. The incidence of annual stroke after TIA is 2-8%.

How well did you know this?
1
Not at all
2
3
4
5
Perfectly
37
Q

Possible causes of lacunar infarction:

1) polycythemia vera
2) Heubner arteritis
3) microatheroma
4) microembolisation

A)  	1st, 2nd and 3rd answers are correct
B)  	1st and 3rd answers are correct
C)  	2nd and 4th answers are correct
D)  	only 4th answer is correct
E)  	all of the answers are correct
A

E) all of the answers are correct

EXPLANATION
Lacunar infarction is caused by occlusion of penetrating small arteries. The most common causes of lacunar infarction are polycythemia vera (disturbance of microcirculation), Heubner arteritis, microatheroma, microembolisation, lipohyalinosis and fibrinoid necrosis of the vessel wall.

How well did you know this?
1
Not at all
2
3
4
5
Perfectly
38
Q

Symptoms caused by occlusion of the inferior branches of middle cerebral artery:

1) contralateral hemianopsia, rarely quadrant anopsia
2) Wernicke- (sensory) aphasia in case of lesion in the dominant hemisphere
3) contralateral transient or mild faciobrachial paresis
4) motor (Broca-) aphasia in case of lesion in the dominant hemisphere

A)  	1st, 2nd and 3rd answers are correct
B)  	1st and 3rd answers are correct
C)  	2nd and 4th answers are correct
D)  	only 4th answer is correct
E)  	all of the answers are correct
A

A) 1st, 2nd and 3rd answers are correct

EXPLANATION
The inferior branches of the middle cerebral artery (MCA) supply the inferior part of the parietal lobe and the superior gyrus of the temporal lobe. Symptoms resulted from occlusion of the inferior branches of MCA can be understood from the functions of the damaged anatomical structures: contralateral hemianopsia or rarely quadrant-anopsia, Wernicke (sensory) aphasia, conductive aphasia, ideomotor apraxia, rarely Gerstmann syndrome in case of lesion in the dominant hemisphere. Contralateral motor symptoms are mild (transient or mild faciobrachial paresis). In case of damage in the non-dominant hemisphere, mild contralateral faciobrachial paresis may be associated with anosognosia, prosopagnosia, dressing and constructive apraxia.

How well did you know this?
1
Not at all
2
3
4
5
Perfectly
39
Q

Characteristic clinical features of the medial medullary lesion (Jackson-syndrome):

1) ipsilateral peripheral hypoglossal lesion
2) contralateral hemiparesis
3) the syndrome is caused by occlusion of the anterior spinal artery and paramedian arteries
4) contralateral tactile and proprioceptive hypesthesia

A)  	1st, 2nd and 3rd answers are correct
B)  	1st and 3rd answers are correct
C)  	2nd and 4th answers are correct
D)  	only 4th answer is correct
E)  	all of the answers are correct
A

E) all of the answers are correct

EXPLANATION
Jackson’s (medial medulla oblongata) syndrome is caused by occlusion of the anterior spinal artery and the paramedian arteries. The symptoms can be understood from the function of the damaged area: ipsilateral peripheral hypoglossal lesion, contralateral hemiparesis (sparing the face). Tactile and proprioceptive hypesthesia may also develop on the contralateral side of the body.

How well did you know this?
1
Not at all
2
3
4
5
Perfectly
40
Q

Clinical features of the lateral medullary lesion (Wallenberg syndrome):

1) dysphagia, dysarthria
2) vertigo, nystagmus
3) algetic- and thermo-hypesthesia on the ipsilateral face and contralateral body
4) Horner’s triad on the ipsilateral side of the lesion

A)  	1st, 2nd and 3rd answers are correct
B)  	1st and 3rd answers are correct
C)  	2nd and 4th answers are correct
D)  	only 4th answer is correct
E)  	all of the answers are correct
A

E) all of the answers are correct

EXPLANATION
Wallenberg (lateral oblongata) syndrome is caused by occlusion of the posterior inferior cerebellar artery. Symptoms include vertigo, nystagmus, gait ataxia, diplopia, dysphagia, dysarthria, ipsilateral Horner triad, and algetic and thermohypesthesia on the ipsilateral face and on the contralateral body side.
How well did you know this?
1
Not at all
2
3
4
5
Perfectly
41
Q

Clinical features of the thalamic lesion:

1) hemihypaesthesia with hemihyperkinesia
2) the basic joints are in flexion position, the interphalangeal joints are in extension postion
3) hyperpathy -burning, shooting pain that is difficult to localise, occurs in attacks and can hardly be influenced by medicines
4) Millard–Gubler syndrome

A)  	1st, 2nd and 3rd answers are correct
B)  	1st and 3rd answers are correct
C)  	2nd and 4th answers are correct
D)  	only 4th answer is correct
E)  	all of the answers are correct
A

A) 1st, 2nd and 3rd answers are correct

EXPLANATION
Since thalamus is related both anatomically and operationally to the hemispherial cortex, and all the afferent systems switch over in the thalamus, thalamic lesion may have several consequences. The ventrolateral nucleus is closely related to the cerebellum, therefore its damage causes hemihypesthesia with hemihyperkinesis (choreoathetosis-like involuntary movements). The so-called “thalamic hand” is also a typical symptom in case of thalamus injury: the basic joints are in flexion while the interphalangeal joints are in extension. Damage of the sensory nuclei often causes “central” or thalamic pain, which can hardly be influenced by medicines. Thalamic pain is characterised by burning, shooting pain that is difficult to localise and occurs in attacks. Thalamus plays a role in the regulation of consciousness as well. Furthermore, thalamus lesion may provoke focal epileptic seizure, and may also cause dementia.

How well did you know this?
1
Not at all
2
3
4
5
Perfectly
42
Q

The following symptoms are always characteristic for central type of lesion:
1) increased abdominal reflexes
2) increased deep reflexes with pyramidal signs
3) anarthria
4) spastic muscle tone
A) 1st, 2nd and 3rd answers are correct
B) 1st and 3rd answers are correct
C) 2nd and 4th answers are correct
D) only 4th answer is correct
E) all of the answers are correct

A

C) 2nd and 4th answers are correct

EXPLANATION
Increased deep-reflexes with pyramidal signs and spastic muscle tone always refer to a central type of lesion.

How well did you know this?
1
Not at all
2
3
4
5
Perfectly
43
Q

Which symptom is not characteristic for the Gerstmann syndrome?

1) left-right confusion
2) finger agnosia
3) acalculia
4) alexia

A)  	1st, 2nd and 3rd answers are correct
B)  	1st and 3rd answers are correct
C)  	2nd and 4th answers are correct
D)  	only 4th answer is correct
E)  	all of the answers are correct
A

D) only 4th answer is correct

EXPLANATIONGerstmann’s syndrome is characterized by left-to-right confusion, finger agnosia, agraphia and acalculia.

How well did you know this?
1
Not at all
2
3
4
5
Perfectly
44
Q

Arteriovenous malformation is suspected in case of the following symptoms and complaints:

1) epileptic seizures
2) supraorbital or occipital bruits
3) headache
4) subarachnoideal hemorrhage

A)  	1st, 2nd and 3rd answers are correct
B)  	1st and 3rd answers are correct
C)  	2nd and 4th answers are correct
D)  	only 4th answer is correct
E)  	all of the answers are correct
A

E) all of the answers are correct

EXPLANATION
If headache and focal or secondarily generalized seizures develop and bruit can be heard in the supraorbital or occipital region, arteriovenous malformation is suspected. In that case the source of the subarachnoidal hemorrhage is probably the vascular malformation.

How well did you know this?
1
Not at all
2
3
4
5
Perfectly
45
Q

Signs of cerebral ischemia with cranial CT scan:

1) fogging phenomenon
2) hypodensity
3) gyral contrast enhancement
4) usually there is no abnormality in the first hours of the ischemic event

A)  	1st, 2nd and 3rd answers are correct
B)  	1st and 3rd answers are correct
C)  	2nd and 4th answers are correct
D)  	only 4th answer is correct
E)  	all of the answers are correct
A

E) all of the answers are correct
EXPLANATION
In cerebral ischemia usually there is no abnormality on cranial CT scan in the first hours of the ischemic event. After 6-8 hours hyperacute ischemic signs could be seen. After 1-2 days clear hypodensity develops in the infracted territory. The hypodensity is mild and has blurred margin in the beginning, however, later it becomes demarcated. Contrast enhancement occurs in the subacute stage, and usually begins at the end of the first week of stroke. Contrast enhancement reaches its maximum at week 2 and 3, and fades over the following weeks. Contrast enhancement usually shows a gyriform pattern on the surface of the brain, but may also occur in the deep parenchyma. It is likely due to a combination of blood brain barrier breakdown, neovascularisation and impaired autoregulation. This phenomenon was previously referred to as ‘luxury perfusion’. Two-three weeks after ischemic stroke, the cortex may show near-normal density (isodense) on native CT, it is the so-called fogging phenomenon. Fogging phenomenon is explained by macrophage invasion, proliferation of capillaries, and sometimes extravasation of blood cells through damaged vessel walls. If in doubt, the administration of IV contrast will demarcate the region of infarction at this stage. Later (week 5-6) the residual swelling passes, and gliosis develops resulting in a region of low density with negative mass effect.

How well did you know this?
1
Not at all
2
3
4
5
Perfectly
46
Q

Indication of cerebral angiography:
1) in urgent cases, when angiography helps in clinical decisions and treatments (e.g. subarachnoideal hemorrhage, local thrombolysis in acute large vessel occlusion)
2) in cerebral contusion to localize the source of the bleeding
3) to plan intra-arterial intervention like intraluminal angioplasty
4) to localise sub- and epidural hematoma
A) 1st, 2nd and 3rd answers are correct
B) 1st and 3rd answers are correct
C) 2nd and 4th answers are correct
D) only 4th answer is correct
E) all of the answers are correct

A

B) 1st and 3rd answers are correct

EXPLANATION
1. and 3. statements are true. Sub- and epidural hematomas are diagnosed with noncontrast cranial CT and there is neither indication for angiography in cerebral contusion.

How well did you know this?
1
Not at all
2
3
4
5
Perfectly
47
Q
Which are the major psychological factors that hinder the effective rehabilitation after stroke?
1)  	emotional lability
2)  	anxiety, anger
3)  	avoiding, aggressive behavior
4)  	mania
A)  	1st, 2nd and 3rd answers are correct
B)  	1st and 3rd answers are correct
C)  	2nd and 4th answers are correct
D)  	only 4th answer is correct
E)  	all of the answers are correct
A

A) 1st, 2nd and 3rd answers are correct

EXPLANATION
The most important psychological characteristics that hinder the early rehabilitation after stroke are emotional lability, dysthimia, anxiety, anger and aggressive behavior. Euphoria is rare.

How well did you know this?
1
Not at all
2
3
4
5
Perfectly
48
Q
Potential consequence(s) of cerebrovascular diseases that hinder successful rehabilitation:
1)  	depression
2)  	schizophreniform reaction
3)  	dementia
4)  	none of them
A)  	1st, 2nd and 3rd answers are correct
B)  	1st and 3rd answers are correct
C)  	2nd and 4th answers are correct
D)  	only 4th answer is correct
E)  	all of the answers are correct
A

B) 1st and 3rd answers are correct

EXPLANATION
Post-stroke depression is relatively common after cerebrovascular diseases (20-50% of the cases). Treating post-stroke depression improves the outcome of rehabilitation. Cognitive decline often appears in cerebrovascular diseases. In vascular dementia, which may also hinder the rehabilitation, the memory and the abstraction are mostly affected

How well did you know this?
1
Not at all
2
3
4
5
Perfectly
49
Q

Transcranial Doppler ultrasound (TCD) examination has to be ordered:
1) to monitor vasospasm after subarachnoid bleeding
2) for emboli detection, if cardiogen stroke is suspected
3) to determine the cerebrovascular reserve capacity with acetazolamide test before carotid endarterectomy
4) in case of vacular lesion in the territory of the middle cerebral artery
A) 1st, 2nd and 3rd answers are correct
B) 1st and 3rd answers are correct
C) 2nd and 4th answers are correct
D) only 4th answer is correct
E) all of the answers are correct

A

E) all of the answers are correct

EXPLANATION
The indications of transcranial Doppler ultrasound (TCD) examination: to monitor vasospasm after subarachnoidal bleeding to examine the flow parameters in intracranial arteries in case of ischemic stroke to detect microemboli, if cardiogenic stroke or unstable carotid plaque is suspected to determine the cerebrovascular reserve capacity with use of acetazolamide test or breath-holding test before carotid surgery

How well did you know this?
1
Not at all
2
3
4
5
Perfectly
50
Q
Which plaque type(s) is (are) considered as source of embolisation on B-mode ultrasound image?
1)  	exulcerated
2)  	haemorrhagic
3)  	heterogeneous
4)  	homogeneous
A)  	1st, 2nd and 3rd answers are correct
B)  	1st and 3rd answers are correct
C)  	2nd and 4th answers are correct
D)  	only 4th answer is correct
E)  	all of the answers are correct
A

A) 1st, 2nd and 3rd answers are correct

EXPLANATION
Regarding the B mode ultrasound imaging, exulcerated, hemorrhagic and heterogeneous plaques are considered as source of embolisation.

How well did you know this?
1
Not at all
2
3
4
5
Perfectly
51
Q

Which of the following examination(s) is (are) needed if cardiogenic stroke is suspected:
1) detailed haematologic examination
2) transthoracic echocardiography (TTE)
3) blood pressure monitoring for 24 hours
4) transesophageal echocardiography (TEE), if TTE couldn’t detect the source of embolisation and if the origin of stroke was unknown in young patient
A) 1st, 2nd and 3rd answers are correct
B) 1st and 3rd answers are correct
C) 2nd and 4th answers are correct
D) only 4th answer is correct
E) all of the answers are correct

A

C) 2nd and 4th answers are correct

EXPLANATION
In case of suspicion of cardiogenic stroke, detailed cardiologic examination is needed, including transthoracic echocardiography (TTE) and transesophageal echocardiography (TEE) if TTE couldn’t identify the source of embolisation and if the origin of stroke was unknown in young patients. Although blood pressure monitoring does not help, Holter ECG is highly recommended if cardiogenic stroke is suspected.

How well did you know this?
1
Not at all
2
3
4
5
Perfectly
52
Q

Match the symptoms with the site of the lesion
A) ideomotor apraxia
B) amusia
C) dysarthria
D) Gerstmann’s syndrome
E) Horner’s syndrome
NEU - 1.124 - caused by a lesion of the pole of the superior temporal gyrus
NEU - 1.125 - caused by bilateral damage of the corticobulbar fibers
NEU - 1.126 - caused by a lesion of the lateral part of the medulla, part of the Wallenberg’s syndrome
NEU - 1.127 - caused by damage of the left supramarginal gyrus in right-handed patients
NEU - 1.128 - caused by a lesion of the left angular gyrus

A

NEU - 1.124 - caused by a lesion of the pole of the superior temporal gyrus - B)
NEU - 1.125 - caused by bilateral damage of the corticobulbar fibers - C)
NEU - 1.126 - caused by a lesion of the lateral part of the medulla, part of the Wallenberg’s syndrome - E)
NEU - 1.127 - caused by damage of the left supramarginal gyrus in right-handed patients - A)
NEU - 1.128 - caused by a lesion of the left angular gyrus-D)

How well did you know this?
1
Not at all
2
3
4
5
Perfectly
53
Q

Match the statements.
A) Speech disturbance and mild right sided hemiparesis developed and resolved within 3 minutes. On cranial CT, lacunar infarctions are seen on the right side.
B) Agraphia with alexia, acalculia, finger agnosia, lower quadrant anopsia evolved, and the symptoms resolved within 2 days.
C) Moderate hemiparesis developed in several hours in the patient admitted with mild left sided hemiparesis, next day hemiplegia evolved and the patient became somnolent and incontinent.
D) Right sided homonymous hemianopsia was detected at the beginning, and pronounced limb ataxia, latent hemiparesis was still present 4 weeks later.
E) Occasionally, the patient complained of severe headache and nausea, which relieved and reoccured, on fundoscopy spontaneous venous pulsation was missing, on the right side brisk deep tendon reflexes were found.NEU - 1.129 - the clinical course may refer to completed stroke
NEU - 1.130 - transient ischemic attack (TIA)
NEU - 1.131 - the clinical course raises suspicion for cerebral tumor rather than stroke
NEU - 1.132 - the symptoms may refer to stroke in the dominant parietal lobe
NEU - 1.133 - the clinical course and syndrome are specific for progressing stroke

A

NEU - 1.129 - the clinical course may refer to completed stroke - D)
NEU - 1.130 - transient ischemic attack (TIA) - A)
NEU - 1.131 - the clinical course raises suspicion for cerebral tumor rather than stroke - E)
NEU - 1.132 - the symptoms may refer to stroke in the dominant parietal lobe - B)
NEU - 1.133 - the clinical course and syndrome are specific for progressing stroke -C)

How well did you know this?
1
Not at all
2
3
4
5
Perfectly
54
Q

Occipital lobe damage causes homonymous hemianopsia, because this symptom is present only in case of occipital lobe injury.
A) both the statement and the explanation are true and a causal relationship exists between them;
B) both the statement and the explanation are true but there is no causal relationship between them;
C) the statement is true, but the explanation is false;
D) the statement is false, but the explanation itself is true
E) both the statement and the explanation are false

A

C) the statement is true, but the explanation is false;

EXPLANATION
After occipital lobe damage homonymous hemianopsia is found, but this sypmtom can also be present in case of optic tract or temporal+parietal lobe (optic radiation) damage.

How well did you know this?
1
Not at all
2
3
4
5
Perfectly
55
Q

The occlusion of the external carotid artery usually does not cause neurological symptoms, because it has no collateral connection with the internal carotid artery.
A) both the statement and the explanation are true and a causal relationship exists between them;
B) both the statement and the explanation are true but there is no causal relationship between them;
C) the statement is true, but the explanation is false;
D) the statement is false, but the explanation itself is true
E) both the statement and the explanation are false

A

C) the statement is true, but the explanation is false;

EXPLANATION
The occlusion of the external carotid artery usually does not cause neurological symptoms. However, there is a collateral connection between the external and the internal carotid arteries.

How well did you know this?
1
Not at all
2
3
4
5
Perfectly
56
Q

In case of a sudden, twinge pain in the frontal and nuchal area during strain, rupture of an intracerebral aneurism should be suspected, because subarachnoidal bleeding could cause cardiac arrythmia.
A) both the statement and the explanation are true and a causal relationship exists between them;
B) both the statement and the explanation are true but there is no causal relationship between them;
C) the statement is true, but the explanation is false;
D) the statement is false, but the explanation itself is true
E) both the statement and the explanation are false

A

B) both the statement and the explanation are true but there is no causal relationship between them;

EXPLANATION
In case of a sudden, twinge pain in the frontal and nuchal areas during strain, intracerebral aneurysm rupture should be suspected. In addition to numerous early and later complications, subarachnoidal bleeding can cause cardiac arrythmia too. Both statements are true, however, there is no connection between them.

How well did you know this?
1
Not at all
2
3
4
5
Perfectly
57
Q

In case of post-stroke depression antidepressant drugs should be used, because depression can inhibit rehabilitation.
A) both the statement and the explanation are true and a causal relationship exists between them;
B) both the statement and the explanation are true but there is no causal relationship between them;
C) the statement is true, but the explanation is false;
D) the statement is false, but the explanation itself is true
E) both the statement and the explanation are false

A

ANSWERA) both the statement and the explanation are true and a causal relationship exists between them;

EXPLANATION
In case of post-stroke depression antidepressant drugs should be used, because depression can inhibit the rehabilitation

How well did you know this?
1
Not at all
2
3
4
5
Perfectly
58
Q

In case of endocarditis lenta anticoagulation therapy is needed, because cerebral embolisation can develop.
A) both the statement and the explanation are true and a causal relationship exists between them;
B) both the statement and the explanation are true but there is no causal relationship between them;
C) the statement is true, but the explanation is false;
D) the statement is false, but the explanation itself is true
E) both the statement and the explanation are false

A

D) the statement is false, but the explanation itself is true

EXPLANATION
In case of endocarditis lenta cerebral embolisation can develop, but anticoagulation therapy is prohibited, because it can increase the chance of cerebral embolisation.

How well did you know this?
1
Not at all
2
3
4
5
Perfectly
59
Q

ECG monitoring in acute phase of stroke is necessary, because stroke may cause cardiac arrythmia.

A) both the statement and the explanation are true and a causal relationship exists between them;
B) both the statement and the explanation are true but there is no causal relationship between them;
C) the statement is true, but the explanation is false;
D) the statement is false, but the explanation itself is true
E) both the statement and the explanation are false

A

A) both the statement and the explanation are true and a causal relationship exists between them;

EXPLANATION
ECG monitoring in the acute phase of stroke is necessary, becasue stroke can cause arrhythmia.

How well did you know this?
1
Not at all
2
3
4
5
Perfectly
60
Q

In case of intracranial arterial occlusion, the lactic acid level raises, NADH/NAD ratio decreases, because superoxide radicals play important role in damaging the mitochondrial membrane.
A) both the statement and the explanation are true and a causal relationship exists between them;
B) both the statement and the explanation are true but there is no causal relationship between them;
C) the statement is true, but the explanation is false;
D) the statement is false, but the explanation itself is true
E) both the statement and the explanation are false

A

B) both the statement and the explanation are true but there is no causal relationship between them;

How well did you know this?
1
Not at all
2
3
4
5
Perfectly
61
Q

The most likely diagnosis is:
After waking up at 6 a.m., a 74-year-old male patient’s wife noticed that her husband couldn’t speak and was unable to move his right extremities. She saw him last at 2 a.m. when her husband went to the toilet and was symptom-free. She called the family doctor, who measured 180/110 mmHg blood pressure and 88/min heart rate. They called the ambulance and arrived at the hospital by 8 a.m. Status: pulmonary emphysema, fundoscopy showed hypertensive retinopathy, right sided central facial palsy, right sided hypotonic, severe hemiparesis predominantly in upper extremities, increased deep tendon reflexes, complete motoric, severe sensoric aphasia. ECG: atrial fibrillation. Electrolytes and blood sugar levels were within the normal ranges.
A) arteriosclerotic encephalopathy
B) ischemic lesion in the territory of the left middle cerebral artery
C) hypertensive encephalopathy
D) Biswanger’s disease

A

B) ischemic lesion in the territory of the left middle cerebral artery

EXPLANATION
Considering the symptoms, the onset of symptoms, and atrial fibrillation in the history, the most likely diagnosis is ishemic lesion in the territirry of the left middle cerebral artery. To verify the diagnosis, the following investigations are needed: cranial CT, duplex scan ultrasound, cardiologic examination with echocardiography. The onset of symptoms is determined by the last time when the patient was symptom-free, in this case 2 a.m. Since atrial fibrillation is known, the heart is the most likely to be the source of embolization, therefore the most probable etiology is cardioembolic.

How well did you know this?
1
Not at all
2
3
4
5
Perfectly
62
Q

Which neurological disease could it be?A 24 years old, primipara woman had strong headache and fever 3 days after delivery. Although the fever was relieved by antipyretic drugs, she became somnolent and developed left sided focal motoric seizure with secondary generalisation. A few hours later the seizures repeated. dministration of 1 mg intravenous clonazepam stopped the seizures. Neurologist found left sided hemiparesis, increased deep tendon reflexes and disturbance of consciousness (somnolence). Prominent scalp edema was also noticed.
A) subarachnoidal bleeding due to aneurysm rupture
B) cerebral embolisation
C) cerebral sinus (venous) thrombosis
D) cerebral hypoxia associated with pneumonia
E) postpartum deep vein thrombosis with cerebral embolization

A

C) cerebral sinus (venous) thrombosis

EXPLANATION
Considering the patient’s hystory, the symptoms, and the progress of the disease, cerebral sinus (venous) thrombosis is suspected. (Based on scalp edema and the other symptoms, superior sagittal sinus thrombosis seems to be the most probable diagnosis.) To verify the diagnosis, cranial CT or MRI with venous CT-angiography or MR-angiography is necessary. (More precise diagnosis can be established by DSA). Searching for the source of infection is also necessary (in this case gynaecological source is the most likely). Treatment: wide spectrum antibiotics, mannitol, therapeutic dose of heparin (even if there is a haemorrhagic transformation).

How well did you know this?
1
Not at all
2
3
4
5
Perfectly
63
Q

Which investigations should be done in order to verify the diagnosis?A 24 years old, primipara woman had strong headache and fever 3 days after delivery. Although the fever was relieved by antipyretic drugs, she became somnolent and developed left sided focal motoric seizure with secondary generalisation. A few hours later the seizures repeated. dministration of 1 mg intravenous clonazepam stopped the seizures. Neurologist found left sided hemiparesis, increased deep tendon reflexes and disturbance of consciousness (somnolence). Prominent scalp edema was also noticed.

1) cranial CT or MRI with venous CTA, or MRA
2) duplex ultrasound examination of cervical arteries
3) search for source of infection (in this case gynecologic source is likely)
4) lumbar puncture, CSF can be bloody

A)  	number 1, 2 and 3 answers are correct
B)  	number 1 and 3 answers are correct
C)  	number 2 and 4 answers are correct
D)  	only the number 4 answer is correct
E)  	all answers are correct
A

ANSWERB) number 1 and 3 answers are correct

How well did you know this?
1
Not at all
2
3
4
5
Perfectly
64
Q

What is the most likely diagnosis?
A 42 years old man lifted a heavy timber made of concrete when he suddenly developed a severe, intolerable headache at the occipital and forehead regions. Hypertension was known in his medical history which was treated by oral medications. He lied down, but his symptoms became worse, furthermore, nausea and photophobia also developed. He became somnolent during transport to neurology ward. Moving the head aggarvated his headache. The neurologist found neck stiffness, increased deep tendon reflexes on both sides. There was no paresis. Blood pressure: 180/100 Hgmm, heart rate: 98/bpm.

A) intracerebral bleeding
B) arteriovenous malformation
C) haemorrhagic infarct
D) subarachnoidal haemorrhage caused by rupture of intracranial aneurysm

A

D) subarachnoidal haemorrhage caused by rupture of intracranial aneurysm

EXPLANATION
Based on the patient’s history and neurological signs, the most likely diagnosis is intracranial aneurysm rupture resulting in subarachnoidal bleeding. In order to verify the diagnosis, cranial CT and cranial CT-angiography or DSA are necessary. CT scan may reveal subarachnoidal bleeding, sometimes the source of bleeding can also be seen. Cranial CT-angiography, or DSA shows the localisation, size and morphology of the aneurysm. Multiple aneurysms may be found, in that case CT (localisation of subarachnoidal bleeding) might help localise the ruptured aneurysm. Vasospasm is a complication of subarachnoidal bleeding, which may cause cerebral ischemia.

How well did you know this?
1
Not at all
2
3
4
5
Perfectly
65
Q

What can be the diagnosis?
A 64 years old, heavy drinker man suddenly collapsed on the street. He lost his consciousness and his face became red. His friend mentioned that the patient had hypertension, however, he did not take any medication. The ambulance team measured 240/120 mmHg blood pressure. Endotracheal intubation was performed because the patient vomited and his respiration was not stable. He had neck stiffness and pain stimulus induced tetraextension in the extremities. His left pupil was dilated and the light reacion on the left side was almost absent. The blood sugar was 14 mmol/L and ECG showed bradycardia.
A) right hemispheric intracerebral bleeding
B) pons haemorrhage
C) cerebellar haemorrhage
D) left hemispheric intracerebral bleeding with mass effect
E) none of them

A

D) left hemispheric intracerebral bleeding with mass effect

EXPLANATION
Based on the symptoms and the positive medical history for untreated hypertension, the most likely diagnosis is left sided intracerebral bleeding with mass effect. Urgent cranial CT is needed. The dilated, and barely reacting pupil on the left side indicates the side of the bleeding (due to the mass effect causing transtentrial herniation the ipsilateral oculomotor nerve is compressed). Tetra-extension to pain stimulus refers to bad prognosis.

How well did you know this?
1
Not at all
2
3
4
5
Perfectly
66
Q

Characteristic features of migraine, except:
A) pulsating pain
B) mostly unilateral
C) physical activity increases the intensity
D) dizziness
E) duration is 4–72 hours

A

D) dizzinessEXPLANATIONThe dissiness is not among the criteria of migraine (not a characteristic feature of migraine) according to the International Headache Society

How well did you know this?
1
Not at all
2
3
4
5
Perfectly
67
Q
What is the prevalence of migraine in adults?
A)  	1–2%
B)  	5–7%
C)  	8–12%
D)  	more, than 20%
A

C) 8–12%

How well did you know this?
1
Not at all
2
3
4
5
Perfectly
68
Q
The onset of primary headaches may be, except:
A)  	childhood
B)  	young age (2–3. decades)
C)  	4th decade
D)  	above 60. years
A

D) above 60. yearsEXPLANATIONHeadache that occurs in the 5th decade or later refers to symptomatic geadache

How well did you know this?
1
Not at all
2
3
4
5
Perfectly
69
Q

Accompanying signs of cluster headache, except:
A) miosis
B) ptosis
C) conjunctival injection
D) paraesthesia on the contralateral side of the headache
E) bradycardia

A

D) paraesthesia on the contralateral side of the headache

EXPLANATIONHemiparaesthesia doesn’t occure in cluster headache

How well did you know this?
1
Not at all
2
3
4
5
Perfectly
70
Q
What is the characteristic nerve conduction velocity of unmyelinitaed pain-transmitting C fibers?
A)  	70–120 m/s
B)  	70–100 m/s
C)  	15–40 m/s
D)  	0.2–2 m/s
A

×ANSWERD) 0.2–2 m/s

How well did you know this?
1
Not at all
2
3
4
5
Perfectly
71
Q
Abdominal lancinating pain is a characteristic sign of:
A)  	spinal cord disease
B)  	tabes dorsalis
C)  	trigeminal-neuralgia
D)  	multiple mononeuritis
A

B) tabes dorsalis

EXPLANATIONLancinating pain is a symptom of lues when granulomatous inflammation infiltrate posterior roots of thoracic spine and ganglion coeliacum and cause pain with sharp quality and very short duration

How well did you know this?
1
Not at all
2
3
4
5
Perfectly
72
Q
What can not be the cause of trigeminal neuralgia?
A)  	idiopathic origin
B)  	multiple sclerosis
C)  	somatoform disturbance
D)  	cancer
A

ANSWERC) somatoform disturbance

EXPLANATIONTrigeminal neuralgia is well localized, is very sharp and is a triggered pain with a duration of a couple of seconds. It can be idiopahic or symptomatic.

How well did you know this?
1
Not at all
2
3
4
5
Perfectly
73
Q

Which one is effective in prophylaxis of cluster headache?A) beta blockers
B) minor analgesics
C) calcium channel blockers
D) tricyclic antidepressants

A

C) calcium channel blockers
EXPLANATION
Verapamil, a calcium channel blocker was proved to be effective for cluster headache prophylaxis.

How well did you know this?
1
Not at all
2
3
4
5
Perfectly
74
Q
It can be effective in the treatment of tension type headache:
A)  	antihistamines
B)  	antihypertensives
C)  	tricyclic antidepressants
D)  	MAO-inhibitors
A

C) tricyclic antidepressants

EXPLANATION
Low dose prolonged use of tricyclic antidepressants is proved for the preventive treatment of tension type headache.

How well did you know this?
1
Not at all
2
3
4
5
Perfectly
75
Q
Which symptom can be caused by unruptured intracranial aneurysm?
A)  	signs of oculomotor nerve lesion
B)  	visual field defect
C)  	ipsilateral recurring headache
D)  	ipsilapteral facial pain
E)  	all of them
A

E) all of them

EXPLANATIONAll of the mentioned complains and symptoms may occur.

How well did you know this?
1
Not at all
2
3
4
5
Perfectly
76
Q
Typical in Wallenberg syndrome:
A)  	contralateral pain
B)  	ipsilateral hypalgesia
C)  	contralateral hemihypalgesia
D)  	ipsilateral facial hypalgesia and contralateral hypalgesia below C2
A

D) ipsilateral facial hypalgesia and contralateral hypalgesia below C2

EXPLANATION
Wallenberg sy is caused by lesion of the dorsolateral medulla oblongata (territory of the posterior inferior cerebellar artery). The decussated spinothalamic and not yet decussated descending trigeminal tract is located at this region, lesion of them results in crossed (alternating) sensory loss (below C2 on the contralateral side of the body and ipsilaterally on the face: algetic, tactile and thermal hypaesthesia).

How well did you know this?
1
Not at all
2
3
4
5
Perfectly
77
Q
Symmetric burning, stabbing pain at the distal part of the extremities, especially on the legs typical for:
A)  	polyneuropathy
B)  	radiculitis
C)  	myositis
D)  	Lambert–Eaton syndrome
A

A) polyneuropathy

EXPLANATIONType and localization of the pain suggests polyneuropathy.

How well did you know this?
1
Not at all
2
3
4
5
Perfectly
78
Q

Typical in migraine, except for:
A) unilateral in more than half of the cases
B) frequently vegetative symptoms
C) EEG always negative
D) ocular symptom in ophtalmoplegic migraine may last for several days
E) frequently aspirin is effective

A

C) EEG always negative

EXPLANATIONA
specific EEG alterations may be observed in migraine patients.

How well did you know this?
1
Not at all
2
3
4
5
Perfectly
79
Q
Shooting pain, dysuria, ataxia associated with pupil disorder, areflexia, and proprioceptiv sensory disturbance suggest the following disease:
A)  	Mixed connective tissue disease
B)  	Tabes dorsalis
C)  	Multiple sclerosis
D)  	Syringomyelia
A

B) Tabes dorsalis

EXPLANATIONThe symptoms described are typical for tabes dorsalis.

How well did you know this?
1
Not at all
2
3
4
5
Perfectly
80
Q
Trigeminal-neuralgia is the least common in the following division of the trigeminal nerve:
A)  	V/2
B)  	V/3
C)  	V/1
D)  	V/2 + V/3
A

C) V/1

EXPLANATION
V/1 localisation is the rarest form in primary (idiopathic) trigeminal neuralgia.

How well did you know this?
1
Not at all
2
3
4
5
Perfectly
81
Q
What is the order of the appearance of herpes zoster symptoms:
A)  	pain-vesicles-pigmentation
B)  	vesicles–pigmentation–pain
C)  	pain–pigmentation–vesicles
D)  	pigmentation– vesicles–pain
E)  	vesicles–pain–pigmentation
A

A) pain-vesicles-pigmentation

EXPLANATION
Radicular pain may precede the diagnostic skin lesions (vesicles) by days.

How well did you know this?
1
Not at all
2
3
4
5
Perfectly
82
Q
Childhood head injury is frequently associated with the following symptom(s):
A)  	vertigo
B)  	epileptic seizure
C)  	headache
D)  	all of them
E)  	none of them
A

D) all of them

EXPLANATION
According to clinical observations childhood head injury is frequently associated with vertigo, epileptic seizure and headache.

How well did you know this?
1
Not at all
2
3
4
5
Perfectly
83
Q
In which localisation can disc herniation cause pain radiating into the thumb?
A)  	CII–CIII
B)  	CIV–CV
C)  	CV–CVI
D)  	CVI–CVII
A

C) CV–CVI

EXPLANATION
Radiating pain in the thumb indicates the lesion of the C6 root. At the cervival region cervical roots exit the spinal column right above the vertebra of the same number as the root itself, so in our example the lesion may be caused by herniation of the CV—VI disc.

84
Q
The following diseases can cause hemifacial pain, except:
A)  	temporomandibular dyskinesia
B)  	trigeminal neuralgia
C)  	tension type headache
D)  	pulpitis (toothache)
A

C) tension type headache

EXPLANATION
Tension type headache usually causes frontal, bilateral headache.

85
Q
The following diseases may be nocturnal (and thus disturbs the sleep), except:
A)  	trigeminal neuralgia
B)  	cluster headache
C)  	migraine without aura
D)  	headache caused by hydrocephalus
A

A) trigeminal neuralgia

EXPLANATIONTrigeminal neuralgia doesn’t disturb sleep, as triggering factors like speaking, chewing and tactile stimuli are usually not present while asleep.

86
Q
The following diseases may cause unilateral ocular pain, except:
A)  	migraine
B)  	cluster headache
C)  	meningeoma of the tentorium
D)  	sinus cavernosus syndroma
E)  	increased intracranial pressure
A

E) increased intracranial pressure

EXPLANATION
Increased intracranial pressure causes diffuse headache

87
Q
What is the prevalance of headache as a primary or associated symptom in brain tumors:
A)  	5–10%
B)  	20–30%
C)  	90–95%
D)  	60–70%
A

ANSWERD) 60–70%

88
Q
The following aura signs may occur in migraine, except:
A)  	hemiparesis
B)  	hemiparaesthesia
C)  	numbness in both hands
D)  	aphasia
A

ANSWERC) numbness in both hands

EXPLANATIONNumbness in both hands is not characteristic for a localised brain dysfunction.

89
Q

Characteristics for subarachnoid haemorrhage:
A) thunderclap headache at the nuchal region
B) the most common neurological finding is spastic hemiplegia or hemiparesis with Babinski sign
C) the most common sign is papilloedema
D) there are no meningeal signs
E) CT scan has no diagnostic value

A

A) thunderclap headache at the nuchal region

EXPLANATIONSubarachnoid hemorrhage is a thunderclap headache mostly in the nuchal region. The other answers cannot be accepted

90
Q

Characteristics for pseudotumor cerebri:
A) It is rare during pregnancy
B) It is rarely associated with endocrinopathy or treatment with medication
C) Vventricules and subarachnoid space are wider than normal
D) Visual accuity can be severely affected
E) Headache is rare

A

D) Visual accuity can be severely affected

EXPLANATIONThe most severe symptom of pseudotumor cerebri is the visual loss. All the other answers are incorrect.

91
Q

Meralgia paraesthetica:
A) its treatment is usually the incision of the fascia lata
B) characterised by burning pain on the surface of the inner part of the thigh
C) often caused by compression of the inguinal ligament
D) more common in women than in men
E) none of them

A

C) often caused by compression of the inguinal ligament

EXPLANATION
Paraesthesia in the territory of lateral cutaneus femoral nerve might be caused by the compression of the nerve.

92
Q
What is characteristic for spondylotic headache?
A)  	usually unilateral
B)  	nuchal pain
C)  	pathological cervical X-ray
D)  	provocated by abnormal head posture
E)  	all of them
F)  	none of them
A

E) all of them

93
Q

Excitation of the sensory cortex causes:
A) paraesthesia
B) hyperaesthesia
C) hyperalgesia

A

ANSWERA) paraesthesia

EXPLANATION
The paraesthesia respects the somatotopy, sometimes it spreads as a sensoric Jacksonian-march

94
Q

Presence of headache requires urgent care, if:
1) it is a thunderclap headache (sudden onset, very severe) at the nuchal region
2) headache is associated with disturbance of consciousness
3) headache is accompanied by neurological signs
4) headache is associated with general symptoms, like fever, or joint pain
A) 1st, 2nd and 3rd answers are correct
B) 1st and 3rd answers are correct
C) 2nd and 4th answers are correct
D) only 4th answer is correct
E) all of the answers are correct

A

E) all of the answers are correct

EXPLANATION
All of the answers are correct, because all of these symptoms refer to symptomatic headache. Urgent examination is needed.

95
Q

Diagnostic criteria of tension type headache:

1) band-like localisation
2) dull and pressing pain
3) mild to moderate intensity
4) physical activity does not aggravate the headache

A)  	1st, 2nd and 3rd answers are correct
B)  	1st and 3rd answers are correct
C)  	2nd and 4th answers are correct
D)  	only 4th answer is correct
E)  	all of the answers are correct
A

E) all of the answers are correct

EXPLANATION
According to the Internacional Headache Society, all of the characteristics listed here belong to the diagnostic criteria of tension headache.

96
Q

Treatment of migraine attacks:

1) minor analgetics
2) ergotamines, dihydroergotamines
3) sumatriptan
4) major analgetics (opioids)

A)  	1st, 2nd and 3rd answers are correct
B)  	1st and 3rd answers are correct
C)  	2nd and 4th answers are correct
D)  	only 4th answer is correct
E)  	all of the answers are correct
A

A) 1st, 2nd and 3rd answers are correct

EXPLANATION
Major analgetic drugs (opioids) should be avoided because the danger of addiction.

97
Q

Headache caused by brain tumor may develop due to:
1) traction or stretch of intracranial vessels
2) increased intracranial pressure
3) dural involvment
4) trigeminal nerve compression
A) 1st, 2nd and 3rd answers are correct
B) 1st and 3rd answers are correct
C) 2nd and 4th answers are correct
D) only 4th answer is correct
E) all of the answers are correct

A

E) all of the answers are correct

EXPLANATION
All of these answers are correct. Tumor may cause headache by traction, stretch, or compression of the pain-sensitive structures, including intracranial vessels, dura, nerves… However, the brain tissue itself is not sensitive to pain because it lacks pain receptors.

98
Q

Characteristics of headache caused by tentorial meningioma:
1) ipsilateral headache in the frontal region
2) the pain radiates to the ipsilateral eye
3) chronic and progressive headache
4) the pain radiates to the ipsilateral shoulder and arm
A) 1st, 2nd and 3rd answers are correct
B) 1st and 3rd answers are correct
C) 2nd and 4th answers are correct
D) only 4th answer is correct
E) all of the answers are correct

A

A) 1st, 2nd and 3rd answers are correct

EXPLANATION
Tentorial meningioma does not cause radiating pain to the shoulder and arm.

99
Q

Acoustic-neurinoma can cause:
1) trismus
2) ear pain
3) trigeminal-neuralgia on the contralateral side
4) trigeminal-neuralgia on the ipsilateral side
A) 1st, 2nd and 3rd answers are correct
B) 1st and 3rd answers are correct
C) 2nd and 4th answers are correct
D) only 4th answer is correct
E) all of the answers are correct

A

D) only 4th answer is correct

EXPLANATIONOnly the fourth answer is correct: among the points listed only the ipsilateral trigeminal neuralgia can be caused by acoustic neurinoma.

100
Q

Which of the following headaches can predict/anticipate rupture of cerebral aneurysm:
1) migraine with aura
2) tension type of headache
3) short-lasting, extremely intense „predictive, anticipating” headache episodes (sentinel headache)
4) cluster headache
A) 1st, 2nd and 3rd answers are correct
B) 1st and 3rd answers are correct
C) 2nd and 4th answers are correct
D) only 4th answer is correct
E) all of the answers are correct

A

B) 1st and 3rd answers are correct

EXPLANATION
Migraine headaches with the same aura symptoms may refer to presence of cerebral aneurysm (although aneurysms are not more frequent in migraine patients than in the normal population), and short-lasting, extremely intense „predictive, anticipating” headache episodes (sentinel headache) may also warn of intracranial aneurysm.

101
Q
Provoking factor(s) is/are in cluster headache:
1)  	yellow cheese
2)  	bending down
3)  	citrus fruits
4)  	beer
A)  	1st, 2nd and 3rd answers are correct
B)  	1st and 3rd answers are correct
C)  	2nd and 4th answers are correct
D)  	only 4th answer is correct
E)  	all of the answers are correct
A

D) only 4th answer is correct

EXPLANATION
In cluster period, alcoholic drinks (like beer) are well-known provoking factors for cluster headache even in small dose

102
Q
Which of the following may cause prosopalgia?
1)  	Glaucoma
2)  	Temporomandibular dyskinesia
3)  	Somatoform disorder
4)  	Allergic rhinitis
A)  	1st, 2nd and 3rd answers are correct
B)  	1st and 3rd answers are correct
C)  	2nd and 4th answers are correct
D)  	only 4th answer is correct
E)  	all of the answers are correct
A

B) 1st and 3rd answers are correct

EXPLANATIONPain caused by temporomandibular dyskinesia may resemble to trigeminal neuralgia. Allergic rhinitis is not painful.

103
Q

Characteristics of thalamic pain:
1) occurs contralateral to the side of the lesion
2) occurs ipsilateral to the side of the lesion
3) severe, intolerable pain
4) mild, dull pain
A) 1st, 2nd and 3rd answers are correct
B) 1st and 3rd answers are correct
C) 2nd and 4th answers are correct
D) only 4th answer is correct
E) all of the answers are correct

A

B) 1st and 3rd answers are correct

EXPLANATION
The thalamic pain occurs contralateral to the side of the lesion, and is severe and intolerable.

104
Q

What are the characteristics of causalgia?
1) It is associated with severe vegetative disorders
2) It is associated with throphic disorders
3) It can be provoked by external stimuli (sound, light, touch)
4) It is a severe, burning pain
A) 1st, 2nd and 3rd answers are correct
B) 1st and 3rd answers are correct
C) 2nd and 4th answers are correct
D) only 4th answer is correct
E) all of the answers are correct

A

E) all of the answers are correct

EXPLANATION
All characteristics listed here are typical for causalgia.

105
Q

Radicular pain can be caused by:

1) extramedullary tumor
2) herpes Zoster
3) disc herniation
4) intramedullary tumor

A)  	1st, 2nd and 3rd answers are correct
B)  	1st and 3rd answers are correct
C)  	2nd and 4th answers are correct
D)  	only 4th answer is correct
E)  	all of the answers are correct
A

A) 1st, 2nd and 3rd answers are correct

EXPLANATION
Intramedullary tumors do not cause radicular pain. All the other diseases may cause damage of a radix resulting in radicular pain.

106
Q

Match the followings:
A) median nerve
B) common peroneal nerve
C) facial nerve
D) ulnar nerve
NEU - 2.97 - Its lesion causes foot drop (steppage gait)
NEU - 2.98 - Its lesion often causes causalgia
NEU - 2.99 - Its injury causes the „Froment’s sign”
NEU - 2.100 - In case of Ramsay-Hunt syndrome this nerve is affected

A

ANSWER
NEU - 2.97 - Its lesion causes foot drop (steppage gait) - B)
NEU - 2.98 - Its lesion often causes causalgia - A)
NEU - 2.99 - Its injury causes the „Froment’s sign” - D)
NEU - 2.100 - In case of Ramsay-Hunt syndrome this nerve is affected -C)

107
Q
Match the different pain syndromes with the potential causes.
A)  	polyneuropathy
B)  	radicular pain
C)  	trigeminal neuralgia
D)  	thalamic pain
NEU - 2.110 - 	extramedullary tumor
NEU - 2.111 - 	hemispheric stroke
NEU - 2.112 - 	chronic alcohol consumption
NEU - 2.113 - 	multiple sclerosis
A
ANSWER
NEU - 2.110 - 	extramedullary tumor	- B)
NEU - 2.111 - 	hemispheric stroke	- D)
NEU - 2.112 - 	chronic alcohol consumption	- A)
NEU - 2.113 - 	multiple sclerosis	- C)
108
Q

Most patients with cluster headache are heavy smokers, because cluster headache is caused by smoking
A) both the statement and the explanation are true and a causal relationship exists between them;
B) both the statement and the explanation are true but there is no causal relationship between them;
C) the statement is true, but the explanation is false;
D) the statement is false, but the explanation itself is true
E) both the statement and the explanation are falsewrong.

A

ANSWERC) the statement is true, but the explanation is false;EXPLANATIONAlthought the most patients with cluster headache are smokers, the etiology of the cluster headache is not known, therefore the second statement is

109
Q

Allergic rhinitis is quite common in migraineurs, therefore antihistamins are effective in prophylactic treatment of migraine.
A) both the statement and the explanation are true and a causal relationship exists between them;
B) both the statement and the explanation are true but there is no causal relationship between them;
C) the statement is true, but the explanation is false;
D) the statement is false, but the explanation itself is true
E) both the statement and the explanation are false

A

B) both the statement and the explanation are true but there is no causal relationship between them;

110
Q

Cluster headache attacks occur with clockwise regularity, therefore disturbance of hypothalamic function is supposed in the cluster period.
A) both the statement and the explanation are true and a causal relationship exists between them;
B) both the statement and the explanation are true but there is no causal relationship between them;
C) the statement is true, but the explanation is false;
D) the statement is false, but the explanation itself is true
E) both the statement and the explanation are false

A

A) both the statement and the explanation are true and a causal relationship exists between them;

EXPLANATION
The hypothalamus is known to contain the „biological clock”, and cluster headache occurs with clockwise regularity. Deep brain stimulation of hypothalamus was reported to be effective in treatment of cluster headache. Both statements are true, and there is a causal relationship between them.

111
Q

Acute stage of viral meningitis is not associated with:
A) common negative clinical findings of cranial CT
B) negative findings on EEG exam
C) cloudy liquor, elevated cell and protein count
D) elevated lymphocyte count in the CSF

A

C) cloudy liquor, elevated cell and protein count

EXPLANATION
In acute viral meningitis, the inflammatory process affects only the soft meninges, the brain tissue is left intact. Therefore, cranial CT or EEG examinations detect no signs of damage to the brain tissue. Clouding of the liquor is not common, but is characteristic of purulent meningitis, also associated with highly elevated cell and protein count. In cases of viral meningitis, the liquor may be clear or colorless, with moderate elevation in cell and protein count.

112
Q

Characteristics of Guillain–Barre syndrome include:
A) segmental demyelinisation may be accompanied by axonal damage
B) axonal lesion does not occur
C) only sensory axonal lesion occurs
D) damage to myelin sheeth is always associated with axonal damage

A

A) segmental demyelinisation may be accompanied by axonal damage

EXPLANATION
In acute Guillain-Barre syndrome, the damage primarily affects the myelin sheath. Still, depending on the aggressiveness of the inflammation, or the prolonged period of convalescence, even the axon may be damaged. Disease damage affects both motor and sensory nerve fibers.

113
Q
Of the following diagnostic procedures, which is most commonly pathological in Multiple Sclerosis?
A)  	eye movement examination
B)  	cranial CT
C)  	EEG
D)  	myelography
A

A) eye movement examination

EXPLANATION
Eye movement disorder is a common characteristic symptom in Multiple Sclerosis. Internuclear ophthamoplegia occurs in case of a pons-mesencephalon lesion, which is rarely caused by other diseases. Convergence of the bulbs may be preserved, but in case of looking sideways, adduction is impaired, and the abducting eye may develop monocular nystagmus. Damage to the vestibulocerebellar pathways or cerebellar lesions may cause nystagmus. Cranial CT using contrast material is able to detect demyelinisational foci, but its sensitivity is lower in contrast with MRI tests. Irregular, circumscribed organic damage or epileptic dysfunction may be present, but is rarely detectable with EEG test. Myelography is used to assess space-occupying lesions in the spinal canal, therefore it cannot detect the demyelinisational foci in Multiple Sclerosis.

114
Q

Diagnostic criteria for NMO (neuromyelitis optica) are:

1) Clinical signs of optic neuritis and/or spinal lesion
2) Presence of aquaporin-4 antibodies in serum
3) Presence of LTM (extensive myelitis overlying several segments) on spinal MR findings
4) Presence of OGP (sign of intrathecal IgG synthesis) in CSF

A)  	answer 1, 2 and 3 correct
B)  	answer 1 and 3 correct
C)  	answer 2 and 4 correct
D)  	only answer 4 correct
E)  	all 4 answers correct
A

A) answer 1, 2 and 3 correct

EXPLANATION
Diagnostic criteria for neuromyelitis optica (NMO) are: 1. longitudinally extensive myelitis with lesion extending over 3 vertebral segments on MRI examination, 2. optic neuritis (neuritis retrobulbaris unilaterally or bilaterally, often recurrent) 3. the presence of antigens against the aquaporin 4 antigen in serum (and/or in liquor). OGP is seldom detectable in the liquor, therefore it does not belong to diagnostic criteria.

115
Q

Characteristic alterations in the CSF in multiple sclerosis:

1) increased glucose content
2) elevated levels of relative immunglobulin- IgG values
3) increased polymorphonuclear leucocyte count in 1/3 of cases
4) increased protein content in app. 1/3 of cases

A)  	answer 1, 2 and 3 correct
B)  	answer 1 and 3 correct
C)  	answer 2 and 4 correct
D)  	only answer 4 correct
E)  	all 4 answers correct
A

C) answer 2 and 4 correct

EXPLANATION
The sugar content in the liquor does not change in Multiple Sclerosis, it is the protein content which changes. Total protein count tends to be within normal limits, but in app. 1/3 of the cases a moderate increase can be observed. Liquor IgG levels are increased, intrathecally synthesized IgG appears in oligoclonal bands (oligoclonal gammopathy, OGP) through electrophoresis. OGP is found in 80-90% of the cases, therefore, it is a significant sign for diagnosis. The appearance of polymorphonuclear leucocytes in the liquor are contraindications for Multiple Sclerosis diagnosis.

116
Q
Common chronic symptoms of multiple sclerosis may be:
1)  	dizziness
2)  	confabulation
3)  	weakness in the extremities, spasticity
4)  	brain oedema
A)  	answers 1, 2 and 3 are correct
B)  	answers 1 and 3 are correct
C)  	answers 2 and 4 are correct
D)  	only answer 4 is correct
E)  	all 4 answers are correct
A

B) answers 1 and 3 are correct

EXPLANATIONMultiple Sclerosis may involve an elated mood, loss of critical sense and cognitive decline due to damage to the frontal lobe. Memory is usually not impaired and confabulation is not a characteristic symptom. The disease does not cause brain oedema or confusion. Damage to central motor neurons, the brain stem and to cerebellum cause characteristic symptoms.

117
Q

Diagnostic specificities of multiple sclerosis (MS):

1) high IgG index in the CSF is specific for MS
2) extended latency values on VEP (visually evoked potentials) are specific for MS
3) gadolinium accumulating white matter lesions detected on MRI are specific for MS
4) dissemination of clinical signs in space and time is prime criterion for the diagnosis

A)  	answers 1, 2 and 3 are correct
B)  	answers 1 and 3 are correct
C)  	answers 2 and 4 are correct
D)  	only answer 4 is correct
E)  	all 4 answers are correct
A

D) only answer 4 is correct

EXPLANATION
There is no single paraclinical test that is specific for MS, therefore, test results must always be evaluated together with the clinical picture. Diagnosis is primarily based on the dissemination of clinical signs in space and time. Intracerebral immunoglobulin synthesis is determined by calculating the IgG index values. Intracerebral IgG synthesis is an important sign of the disease, but not specific for MS. Triggered response reaction tests are used to detect subclinical lesions, but are not disease specific, either. Native MRI tests are similar from that respect, and are used to test for the demyelinisating character of the foci, and may also indicate the multiplied nature of the process, while possible gadolinium enhancement may indicate disease activity. These deviations on MR findings do not occur only with MS.

118
Q
Spasticity in multiple sclerosis may be reduced using the following medications:
1)  	dantrolen
2)  	tizanidine
3)  	botulinumtoxin
4)  	baclofen
A)  	answer 1, 2 and 3 correct
B)  	answer 1 and 3 correct
C)  	answer 2 and 4 correct
D)  	only answer 4 correct
E)  	all 4 answers correct
A

C) answer 2 and 4 correct

EXPLANATION
Damage to pyramidal pathways leads to the unpleasant increase in spastic muscle tone activity in Multiple Sclerosis. It may cause pain and impair physical performance. Patient response to this condition varies, therefore, the administration of baclofen or tizanidine may be advised.

119
Q

In multiple sclerosis, introduction of immunomodulatory treatment should be indicated:

1) in relapsing-remitting clinical form, during the active phase
2) in the primary, progressive form
3) in the early stage, until the patient retains mobility
4) in a secondary, progressive form, over age 50

A)  	answer 1, 2 and 3 correct
B)  	answer 1 and 3 correct
C)  	answer 2 and 4 correct
D)  	only answer 4 correct
E)  	all 4 answers correct
A

B) answer 1 and 3 correct

EXPLANATION
Immunomodulating treatment in the relapse-remission form of MS significantly reduce relapse frequency, and moderates the degree of tissue damage. Its effectiveness in other forms of the disease have not been satisfactorily elucidated. Its use is usually not indicated in patients with severe deterioration of movement (who lost individual walking capacity).

120
Q
Based on the signs and symptoms, the case can be:A 48 years old male patient, who regularly consumes alcohol, became restless in the morning on the day of the hospital admission, and suffered a Grand Mal Seizure. He was scheduled for a surgery due to colon cancer half a year ago. Three days prior to the admission, he experienced a fever and his temperature rose to 38 ° C. Upon examination, a mild stiffness in the neck, a horizontal nystagmus (while looking in both directions), and a moderate limb and trunk ataxia were discovered. He was slightly somnolent.
A)  	purulent meningitis
B)  	brain abscess
C)  	brain tumor of metastatic origin
D)  	all of the above
E)  	none of the above.
A

D) all of the above

EXPLANATION
People, who regularly consume alcohol, are more susceptible to infections, and they are more likely to develop central nervous system complications. A high fever, stiffness in the neck, epileptic attacks and an impaired consciousness point towards purulent meningitis. Although physical examinations describing organic damage are not documented, largely due to the disorientation, the possibility of brain abscess of space occupying nature, accompanying febrile illnesses, should be maintained. Intracranial space-occupying processes, apart from disorientation, too, can cause mild stiffness in the neck, or epileptic attacks. One of the common sources of metastatic brain tumors is colon carcinoma. This, too, needs to be taken into account. Given that there are signs of intracranial hypertension, acute imaging (CT or MRI) tests should be arranged. In the case of purulent meningitis, the cerebrospinal fluid cell count significantly increased (10000/3 or up to a multiple of this). We also find a significant increase in CSF total protein, which can be double of the normal amount, or even greater. The examination of the cerebrospinal fluid may have diagnostic value in the case of both meningitis, brain abscess and metastatic cancer. Lumbar puncture should be avoided in the case of papilledema. A cranial CT and MRI play an important role in determining both brain abscess and metastatic brain tumors

121
Q

The brain contusion almost always involves a loss of consciousness, and no abnormalities are visible on the CT scan and the CSF is often bloody.
A) true
B) false

A

B) false

EXPLANATION
In the case of contusio cerebri (cerebral contusion), the patient loses consciousness. However, in rare cases with localized force, loss of consciousness does not necessarily occur. The brain is damaged, which can be detected by cranial CT or MRI at the early stage. During cerebral contusion, the CSF, also at an early stage, can be bloody or xantochrom.

122
Q

The number of cells increases and protein content decreases in the CSF during encephalitis.
A) true
B) false

A

B) false

EXPLANATION
There could be a mild degree of cell growth (50-100/3) in encephalitis, as a larger number of cells is considered not typical. Similarly, the protein level of the CSF can be normal or slightly increased.

123
Q

Cauda equina syndrome is not pyramidal.
A) true
B) false

A

A) true

EXPLANATION
The cauda equina is a bundle of motor and sensory roots associated with the lumbar and sacral segments. Accordingly, any damage to the cauda does not create any pyramid symptoms signaling the abuse of the central motor neuron.

124
Q

The visual evoked potential (VEP) is still intact in most patients suffering from multiple sclerosis.
A) true
B) false

A

B) false

EXPLANATION
Multiple sclerosis often affects the optic nerve, and in many cases, the first sign of the disease is optic neuritis. The visual evoked potential (VEP) may indicate (subclinical) optic nerve damage in patients, who did not voice strong complaints.

125
Q

Generally, a vertical nystagmus can be observed in the case of vertigo of peripheral origin.
A) true
B) false

A

B) false
EXPLANATION
Vertical nystagmus in the mesencephalon is a sign of brainstem damage. It does not occur in the case of peripheral vestibular disease.

126
Q

With appropriate treatment and care, the life expectancy of patients with multiple sclerosis is only a few years less than that of the general population of the same age.
A) true
B) false

A

A) true

EXPLANATION
By providing regular monitoring by specialists, appropriate medical treatment, counseling and social care, the disease shortens the life of the patient by only a few years.

127
Q
Oral and manual automatisms are characteristics for:
A)  	absence
B)  	complex partial seizures
C)  	generalized tonic-clonic seizures
D)  	sensomotor Jacksonian seizure
E)  	tetany
A

B) complex partial seizures

EXPLANATION
Oral automatisms (lip smacking, chewing, or swallowing) and manual automatisms (pillrolling, finger rubbing) are the main characteristics of complex partial seizures which usually originate from the temporal lobe.
128
Q
Antiepileptic drug with a specific indication:
A)  	TSH
B)  	L-dopa
C)  	acetylcholine
D)  	ACTH
E)  	digitalis
A

D) ACTH

EXPLANATION
One of the most severe epileptic encephalopathies in infants is the West syndrome consisting of typical seizure form (BNS seizure or infantile spasm) and typical EEG sign (hypsarrhythmia). „Classical” antiepileptic drugs with exception of vigabatrin are less effective in this syndrome. Conversely, ACTH and steroid treatment can be very effective, may result in compete seizure freedom.

129
Q

NOT a characteristic of convulsive syncope:

A)  	open eyes
B)  	jerking
C)  	sweating
D)  	pallor
E)  	lateral tongue biting
A

E) lateral tongue biting

EXPLANATION
Grand mal (generalized tonic-clonic seizure) should be differentiated from convulsive syncope and the psychogenic non-epileptic seizure (pseudoseizure, „hysteric” seizure). Convusiv syncope – like any syncope – can precede by fainting sensation, pallor, or sweating. Conversely, during the unconsciousness, a brief tonic phase, myoclonic or clonic jerking can occur. In contrast to grand mal, convulsive syncope never begins with a tonic phase (it starts with a loss of muscle tone). Tongue biting can also occur in convulsive syncope, but it never affects the lateral part of tongue, rather the tip of tongue (apical tongue biting).
130
Q
The most frequently used type of epilepsy surgery is:
A)  	parietal lobectomy
B)  	temporal lobectomy
C)  	callosotomy
D)  	hemispherectomy
E)  	multiple subpial transection
A

B) temporal lobectomy

EXPLANATION
Most adult patients with pharmacoresistant epilepsy have temporal lobe epilepsy. The most common epilepsy surgery procedure is the temporal lobectomy.

131
Q

Interictal EEG sign suggesting epilepsy:

A)  	vertex potential in sleep
B)  	3 Hz spike-and-wave complex
C)  	6–14 Hz positive discharges
D)  	none of them
E)  	all of them
A

B) 3 Hz spike-and-wave complex

EXPLANATION
The vertex potential on the EEG is characteristic to the light sleep. This is a physiological phenomenon. The 6-14 Hz positive spikes are normal (physiological) variations of the EEG. A 3Hz spike-and wave complex is the typical EEG sign of some genetically determined epilepsy forms, such as idiopathic generalized epilepsy including absence epilepsy.

132
Q

A 55 year-old man was admitted to the emergency department due to the first epileptic seizure in his life. Neurological examination revealed a mild left-sided (central) facial paresis and left-sided hemiparesis. MRI showed a space-occupying lesion in the right frontal lobe. What is the most probable diagnosis?

A) occlusion of the right-sided internal carotid artery.
B) medulloblastoma
C) cerebellar astrocytoma
D) provoked seizure due to alcohol withdrawal
E) glioma

A

E) glioma

EXPLANATION
Brain tumor is the underlying cause of 20-30% of adult epilepsies. Epileptic seizure is the first clinical manifestation in 1/3rd of brain tumor cases.

133
Q
Prevalence of epilepsy in industrialized countries:
A)  	0.04-0.05%
B)  	0.02%
C)  	0.5–1%
D)  	5–6%
E)  	10–12%
F)  	8–10%
A

C) 0.5–1%

EXPLANATION
The incidence of epilepsy in industrial high-income countries is 0.04-0.1%, while the prevalence is 0.5-1%. This prevalence rate is about 3-6 times higher (ca. 3%) in some countries with limited resources due to endemic cysticercosis

134
Q

Seizures with „jamais vu” usually occur in:

A) frontal lobe epilepsy
B) temporal lobe epilepsy
C) parietal lobe epilepsy
D) occipital lobe epilepsy

A

B) temporal lobe epilepsy

EXPLANATION
The ”jamais vu” phenomenon (I have never seen, I have never been in this situation) is a typical epileptic dysfunction of the temporo-limbic system: a specific aura type in temporal lobe epilepsy.

135
Q

Postictal exhaustion of the brain can manifest in:

1) tenebrosity
2) Todd-paresis
3) terminal sleep
4) aphasia
A) 1., 2. and 3. answers are correct
B) 1. and 3. answers are correct
C) 2. and 4. answers are correct
D) only 4. answer is correct
E) all 4 answers are correct

A

ANSWER
E) all 4 answers are correct

EXPLANATION
After an epileptic seizure, the involved cerebral structures become exhausted. This can manifest in clinical symptoms. General exhaustion after grand mal can appear as postictal sleep (called terminal sleep: a postictal sopor-like state) or in postictal confusion. Due to exhaustion of limbic structures, postictal confusion can also emerge after temporal lobe seizures. Clonic seizures can be followed by transient paresis (Todd paresis). Todd paresis is caused by postictal exhaustion of the cortical motor centers. The same mechanism is responsible for postictal aphasia appearing after left-sided temporal lobe seizures.

136
Q

An option in epilepsy treatment:

1) vagus nerve stimulation
2) ketogenic diet
3) benzodiazepines
4) deep brain stimulation

A)  	1., 2. and 3. answers are correct
B)  	1. and 3. answers are correct
C)  	a 2. and 4. answers are correct
D)  	only 4. answers is correct
E)  	all 4 answers are correct
A

E) all 4 answers are correct

EXPLANATION

Chronic deep brain stimulation of the anterior nucleus of the thalamus (ANT-DBS) and the vagus nerve stimulation (VNS) are effective neuromodulation therapies of drug-resistant epilepsy. The ketogenic diet can be highly effective in some childhood epilepsy syndromes. Benzodiazepines are the first-choice drugs in status epilepticus, moreover, some per os benzodiazepines (clobazam, clonazepam) can also be used in chronic epilepsy treatment.

137
Q
Hepatic enzyme inducers:
1)  	carbamazepine
2)  	levetiracetam
3)  	diphenylhydantoin
4)  	gabapentin
A)  	1., 2. and 3. answers are correct
B)  	1. and 3. answers are correct
C)  	a 2. and 4. answers are correct
D)  	only 4. answers is correct
E)  	all 4 answers are correct
A

B) 1. and 3. answers are correct

EXPLANATION
Both carbamazepine and diphenylhydantoin (phenytoin) are enzyme inducers. This can be problematic in case of drug combinations due to drug-to-drug interactions.

138
Q
Effective in status epilepticus:
1)  	diazepam iv.
2)  	clonazepam iv.
3)  	propofol iv.
4)  	diphenylhydantoin iv.
A)  	1., 2. and 3. answers are correct
B)  	1. and 3. answers are correct
C)  	2. and 4. answers are correct
D)  	only 4. answer is correct
E)  	all 4 answers are correct
A

E) all 4 answers are correct

EXPLANATION
The most effective first-line treatment of status epilepticus is the intravenous administration of benzodiazepines. If intravenous administration is not possible, then rectal diazepam or buccal midazolam is a reasonable alternative. If the status epilepticus continues despite benzodiazepine treatment, then intravenous diphenylhydantoin, valproate, or levetiracetam should be administrated. In refractory status epilepticus, artificial coma should be induced by propofol, short-acting barbiturates, or midazolam.

139
Q
Epileptic seizure can be provoked by:
1)  	some psychotropic drugs
2)  	sleep deprivation
3)  	Cocoa consumption
4)  	alcohol withdrawal
5)  	infection with fever
A)  	all of them are true
B)  	Answer 2, 4, 5 are true
C)  	Answer 2, 3, 4, 5 are true
D)  	Answer 1, 2, 4 are true
E)  	Answer 1, 2, 4, 5 are true
A

E) Answer 1, 2, 4, 5 are true

EXPLANATIONCocoa consumption does not cause seizures. Conversely, sleep deprivation, alcohol withdrawal, some drugs, and fever can provoke — in some susceptive individuals — epileptic seizures (called provoked seizures).

140
Q

Generalized tonic-clonic seizures:

1) tetany
2) provoked epileptic seizures
3) cataplexy
4) seizures of eclampsia in pregnancy
A) Answer 1, 2, 3 are true
B) Answer 2, 3, 4 are true
C) Answer 2 and 4 are true
D) Answer 1 and 4 are true
E) all of them are true

A

C) Answer 2 and 4 are true

EXPLANATION
Eclampsia is a disorder of pregnancy characterized by epileptic seizures in the setting of pre-eclampsia. Tetany is caused by hypocalcaemia and not an epileptic seizure. Cataplexy (characterized by sudden loss of muscle tone) is not an epileptic seizure but one of the symptoms of narcolepsy .

141
Q

Pair the matching items!

A)  	epilepsia partialis continua
B)  	Jacksonian seizure (Jacksonian march)
C)  	absence
D)  	tonic seizure
E)  	sleep attack
NEU - 4.41 - 	Lennox–Gastaut syndrome
NEU - 4.42 - 	3-Hz spike-and-wave complex on the EEG
NEU - 4.43 - 	focal status epilepticus
NEU - 4.44 - 	focal seizure
NEU - 4.45 - 	narcolepsy
A

NEU - 4.41 - Lennox–Gastaut syndrome - D)
NEU - 4.42 - 3-Hz spike-and-wave complex on the EEG - C)
NEU - 4.43 - focal status epilepticus - A)
NEU - 4.44 - focal seizure - B)
NEU - 4.45 - narcolepsy - E)

142
Q
Diagnosis:
A 18-year-old young man had grand mal seizures since age 14. No provoking factors were revealed. The patient reported that he also had involuntary jerking in the mornings.
A)  	focal epilepsy
B)  	juvenile myoclonic epilepsy
C)  	chorea major
D)  	psychogenic epilepsy
A

B) juvenile myoclonic epilepsy

EXPLANATION
Juvenile myoclonic epilepsy is one of the most frequent epilepsy syndromes in adulthood: its prevalence is ca. 0.1%. It is characterized by grand mal seizures and myoclonic jerks (usually on awakening). In 40% of patients, absence seizures can also occur. The EEG shows a 3-Hz spike-and-wave pattern. This is a genetic epilepsy, consequently, the MRI shows no epileptogenic lesion.

143
Q

Typical sign(s) of Huntington’s disease:

A) blepharospasm
B) areflexia
C) pyramidal signs
D) dementia

A

D) dementia

EXPLANATION
In Huntigton-chorea beside of the hyperkinesia, the subcortical type of dementia is the most typical neuro-psychiatrical sign.

144
Q

The typical sign of the Wilson’s disease is:
A) dystonia
B) doll’s eye phenomenon
C) signs of meningeal irritation
D) the lack of the abdominal skin reflex

A

A) dystonia

EXPLANATION
In Wilson’s disease there could be any type of dyskinesias, we can see first of all torsinon type of dystonia.

145
Q
The prevalence of the Parkinson’s disease is approximately:
A)  	1–2 patients per 100 000 people
B)  	5–10 patients per 100 000 people
C)  	10–20 patients per 100 000 people
D)  	100–200 patients per 100 000 people
A

D) 100–200 patients per 100 000 people

EXPLANATION
The prevalence of Parkinson’s disease is the same all over the World, under 65 years there are 100-200 patients in 100 000 people.

146
Q

The signs of the Parkinson’s disease:

1) rigidity
2) ataxic gait
3) hypersalivation
4) saccadic speech

A)  	1st, 2nd and 3rd answers are correct
B)  	1st and 3rd answers are correct
C)  	2nd and 4th answers are correct
D)  	only 4th answer is correct
E)  	all of the answers are correct
A

B) 1st and 3rd answers are correct

EXPLANATION
According to the international diagnostic criteria, the rigor and salivation are typical for Parkinson’s disease among the answers listed here.

147
Q

Ballism:

1) may be a symptom of Parkinson’s disease
2) hypertonic hyperkinesis
3) lesion of the fronto-ponto-cerebellar tract is etiological
4) caused by the lesion of sub thalamic nucleus
A) 1st, 2nd and 3rd answers are correct
B) 1st and 3rd answers are correct
C) 2nd and 4th answers are correct
D) only 4th answer is correct
E) all of the answers are correct

A

D) only 4th answer is correct

EXPLANATIONThe lesion of the subthalamic nucleus results in contralateral (hemi)ballism.

148
Q

There is no tremor during voluntary treatment in Parkinson’s disease, because it is ceased by the cerebellar inhibitory pathways.

A) both the statement and the explanation are true and a causal relationship exists between them;
B) both the statement and the explanation are true but there is no causal relationship between them;
C) the statement is true, but the explanation is false;
D) the statement is false, but the explanation itself is true
E) both the statement and the explanation are false

A

C) the statement is true, but the explanation is false;

EXPLANATIONThere is no intention tremor in Parkinson’s disease, but not because of the activation of inhibitory cerebellar tracts. Intention tremor is caused by the injury of another (dentato-rubro-thalamic) tract.

149
Q

Which symptom is not characteristic for Amyotrophic lateral sclerosis?
A) Babinski-reflex (extensor plantar reflex)
B) atrophy of the small hand muscles
C) fasciculation
D) dysphagia
E) paraesthesia

A

E) paraesthesia

EXPLANATION
Paraesthesia: not a sign of ALS. Sensory system is not affected in this disease. ALS is a disease of both the lower and upper motor neurons (with the degeneration of the corticospinal tract).

150
Q

When to suspect polymyositis?
A) numbness with paresis
B) paresis starts in the proximal limb muscles
C) normal erythrocyte sedimentation rate
D) paresis in the proximal limb muscles along with fever and pain

A

D) paresis in the proximal limb muscles along with fever and pain

EXPLANATION
Polymyositis: generalized, subacutely progressing, mostly symmetric, chronic disease of the skeletal muscles with histologic signs of inflammation. Weakness of truncal and proximal muscles of the extremities associated with pain and fewer. (Pathological immunological processes in the background.)

151
Q

Which symptom is not provoked by muscarin?

A)  	nausea
B)  	salivation
C)  	fasciculation
D)  	bronchial secretion
E)  	vertigo
A

C) fasciculation

EXPLANATION
Fasciculation: it’s a nicotinergic symptom.

152
Q

Which is not characteristic for Duchenne muscular dystrophy?
A) X-linked recessive inheritance
B) loss of ambulation at 25–30 years of age
C) selective damage of the muscles
D) markedly high CK levels at early phases
E) absence of dystrophin in skeletal muscles

A

B) loss of ambulation at 25–30 years of age

EXPLANATION
Duchenne disease: X-linked muscular dystrophy of recessive inheritance (severe form of dytrophia musculorum progressiva). Progressive weakness of the truncal, limb girdle, upper arm, gluteal, hip flexor and knee extensor muscles. The onset is in the infancy, patients usually die before 20-year of age.

153
Q
Frequent symptom in ALS:
A)  	dementia
B)  	agnosia
C)  	problems with urination
D)  	all
E)  	none
A

E) none

EXPLANATION
ALS is a disease of both the lower and upper motor neurons (with the degeneration of the corticospinal tract). Sometimes dementia is associated with ALS (not frequently). In these cases significant neuronal damage and gliosis can be observed in the premotor gyrus and the temporal lobe.

154
Q

A 25-year old female complains about gait disturbance for two months. Walking on stairs is most problematic. Laboratory tests reveal increased ESR, CK and gamma-globulin levels. What is the most likely diagnosis?
A) poliomyelitis
B) polymyositis
C) Coxsackie-infection
D) autosomal recessive muscular dystrophy
E) myasthenia gravis

A

B) polymyositis

EXPLANATION
Polymyositis: generalized, subacutely progressing, mostly symmetric, chronic disease of the skeletal muscles with histologic signs of inflammation. Weakness of truncal and proximal muscles of the extremities associated with pain and fewer. (Pathological immunological processes in the background.) Laboratory tests reveal increased ESR, gamma-globulin and markedly elevated CK levels.

155
Q
The typical liquor of acute Guillain-Barré syndrome is characterised by:
1)  	increased protein value
2)  	decreased protein value
3)  	normal cell count
4)  	increased cell count
A)  	1st, 2nd and 3rd answers are correct
B)  	1st and 3rd answers are correct
C)  	2nd and 4th answers are correct
D)  	only 4th answer is correct
E)  	all of the answers are correct
A

B) 1st and 3rd answers are correct

EXPLANATION
Typical liquor finding: cell-protein dissociation (the total protein content of the CSF is increased, while the cell count is normal or only slightly elevated).

156
Q
The neurological status of the Guillain-Barré syndrome may include:
1)  	paraparesis
2)  	hypo- or areflexia
3)  	hypaesthesia
4)  	diplegia facialis
A)  	1st, 2nd and 3rd answers are correct
B)  	1st and 3rd answers are correct
C)  	2nd and 4th answers are correct
D)  	only 4th answer is correct
E)  	all of the answers are correct
A

E) all of the answers are correct

EXPLANATION
Acute Guillain-Barré syndrome (GBS) may include all of the symptoms listed here. GBS begins typically with distal type of paraesthesia and hypotonic paresis that may ascend in the following days. Lower cranial nerve symptoms are present in almost half of the GBS cases (dysphagia, diplegia facialis).

157
Q

The differential diagnosis of amyotrophic lateralsclerosis (ALS) includes the following disease(s):
1) multiple sclerosis
2) multifocal motoric neuropathy
3) compression of the cervical spinal cord
4) cervical spondylosis
5) funicular myelosis
A) 1st, 2nd and 3rd answers are correct
B) 1st, 3rd and 4th answers are correct
C) 2nd, 3rd and 4th answers are correct
D) 2nd, 3rd and 5th answers are correct
E) 2nd, and 4th answers are correct
F) 3rd, 4th and 5th answers are correct

A

C) 2nd, 3rd and 4th answers are correct

EXPLANATION
ALS is a disease of the lower and upper motoneurons with degeneration of the corticospinal tract. Those disorders should be considered in differential diagnosis, which causes purely motor symptoms including paralysis, fasciculation, muscle atrophy, and muscle tone changes. MS: can be differentiated by sensory and vegetative symptoms that are associated with MS. Multifocal motor neuropathy: causes purely motoric symptoms, therefore it should be considered in the differential diagnosis. Cervical root and cervical spinal cord compression (e.g. cervical disc herniation) lead to pyramidal lesion and spasticity (upper motoneuron signs), as well as fasciculations (lower motoneuron sign caused by radicular compression). It should also be considered in the differential diagnosis. Cervical spondylosis, discopathy may also cause fasciculation, paresis, atrophy, therefore it should be differentiated from ALS too. Funicular myelosis: can easily be differentiated by disturbance of deep-sensation.

158
Q

The Tensilon (edrophonium) test is used to differentiate myasthenic and cholinergic crises because administration of edrophonium improves the symptoms dramatically in myasthenic crisis

A) both the statement and the explanation are true and a causal relationship exists between them;
B) both the statement and the explanation are true but there is no causal relationship between them;
C) the statement is true, but the explanation is false;
D) the statement is false, but the explanation itself is true
E) both the statement and the explanation are false

A

A) both the statement and the explanation are true and a causal relationship exists between them;

EXPLANATION
Tensilon® (edrophoniume): fast and short acting cholinesterase inhibitor, which improves both the clinical and electrophysiological myasthenic symptoms, while the cholinergic crisis is not worsened. Thus, the cholinergic and myasthenic crisis can be differentiated by the edrophonium test.

159
Q
Possible complications:
A 28-year-old man presented with progressing lower extremity weakness and ascending numbness that started about a week ago. He reported that he had a strong diarrhea three weeks ago.
1)  	paralysis of respiratory muscles
2)  	cranial nerve lesion
3)  	deep vein thrombosis
4)  	vegetative disturbances
A)  	1st, 2nd and 3rd answers are correct
B)  	1st and 3rd answers are correct
C)  	2nd and 4th answers are correct
D)  	only 4th answer is correct
E)  	all of the answers are correct
A

E) all of the answers are correct

EXPLANATION
He has Guillan Barre.
Each of the options listed can occur: a) respiratory muscle paralysis: due inflammation of the cervical roots, b) cranial nerve lesion: in 50% of the cases, lower cranial nerve symptoms can be observed c) lower limb deep vein thrombosis: due to immobilisation d) vegetative disorder: orthostatic hypotension, sweating, rarely urinary and bowel incontinence.

160
Q

In sural nerve biopsy we would find the following:
A 28-year-old man presented with progressing lower extremity weakness and ascending numbness that started about a week ago. He reported that he had a strong diarrhea three weeks ago.
1) neutrophil granulocyte infiltration
2) segmental demyelination
3) intact nerve
4) axonopathy may occur
A) 1st, 2nd and 3rd answers are correct
B) 1st and 3rd answers are correct
C) 2nd and 4th answers are correct
D) only 4th answer is correct
E) all of the answers are correct

A

C) 2nd and 4th answers are correct

EXPLANATION
Guillan Barrè.
Sural nerve biopsy shows segmental demyelination, but rarely axonal damage can also be found.

161
Q
Typical symptoms of transient global amnesia, EXCEPT:
A)  	loss of anterograde memory
B)  	the patient is alert
C)  	shorter than 24 hours
D)  	cortical blindness
E)  	unknown cause
A

D) cortical blindness

162
Q

Cortical lesion is probable in case of stroke in the territory of the internal carotid artery if:
A) hemiparesis with dominance in the facial and brachial area
B) if the severity of the paresis is similar on the upper and lower limb
C) visual field defect is also seen
D) permanent vertigo develops

A

A) hemiparesis with dominance in the facial and brachial area

163
Q

What are the important side effects of ticlopidine?
A) thrombin time will be doubled
B) neutropenia, especially in the first 3 months
C) thrombocyte aggregation inhibition
D) vertigo

A

B) neutropenia, especially in the first 3 months

164
Q

Possible causes of subarachnoid hemorrhage:
1) rupture of bridging veins
2) cerebral contusion
3) immunogenic vasculitis
4) rupture of arteriovenous malformation
A) 1st, 2nd and 3rd answers are correct
B) 1st and 3rd answers are correct
C) 2nd and 4th answers are correct
D) only 4th answer is correct
E) all of the answers are correct

A

C) 2nd and 4th answers are correct

165
Q
Symptoms of hypertensive encephalopathy:
1)  	epileptic seizures
2)  	vomiting
3)  	disturbance of consciousness
4)  	cerebral edema
A)  	1st, 2nd and 3rd answers are correct
B)  	1st and 3rd answers are correct
C)  	2nd and 4th answers are correct
D)  	only 4th answer is correct
E)  	all of the answers are correct
A

E) all of the answers are correct

166
Q

Characteristic clinical features of the basilar artery occlusion:
1) locked-in syndrome
2) severe gnostic dysfunction
3) sudden onset of coma, tetra-pyramidal signs
4) apraxia
A) 1st, 2nd and 3rd answers are correct
B) 1st and 3rd answers are correct
C) 2nd and 4th answers are correct
D) only 4th answer is correct
E) all of the answers are correct

A

B) 1st and 3rd answers are correct

167
Q

The most important symptoms of cerebral aneurysm rupture:
1) sudden onset of a severe, throbbing nuchal or frontal headache
2) gradually developing headache, dizziness
3) neck stiffness
4) focal neurological signs are already present at the onset of the disease
A) 1st, 2nd and 3rd answers are correct
B) 1st and 3rd answers are correct
C) 2nd and 4th answers are correct
D) only 4th answer is correct
E) all of the answers are correct

A

B) 1st and 3rd answers are correct

168
Q
What can cause septic aneurysm in the middle cerebral artery?
1)  	dissection
2)  	bacterial pneumonia
3)  	viral infection
4)  	endocarditis
A)  	1st, 2nd and 3rd answers are correct
B)  	1st and 3rd answers are correct
C)  	2nd and 4th answers are correct
D)  	only 4th answer is correct
E)  	all of the answers are correct
A

C) 2nd and 4th answers are correct

169
Q
Arteriovenous malformation is suspected in case of the following symptoms and complaints:
1)  	epileptic seizures
2)  	supraorbital or occipital bruits
3)  	headache
4)  	subarachnoideal hemorrhage
A)  	1st, 2nd and 3rd answers are correct
B)  	1st and 3rd answers are correct
C)  	2nd and 4th answers are correct
D)  	only 4th answer is correct
E)  	all of the answers are correct
A

E) all of the answers are correct

170
Q

Which of the following statements is (are) typical for intracranial hemorrhage?
1) appears mostly daytime, during physical activity
2) appears during sleep, symptoms are recognized after waking up
3) sudden onset, rapid, progressive disturbance of consciousness
4) TIA in the past medical history
A) 1st, 2nd and 3rd answers are correct
B) 1st and 3rd answers are correct
C) 2nd and 4th answers are correct
D) only 4th answer is correct
E) all of the answers are correct

A

B) 1st and 3rd answers are correct

171
Q

Signs of cerebral ischemia with cranial CT scan:
1) fogging phenomenon
2) hypodensity
3) gyral contrast enhancement
4) usually there is no abnormality in the first hours of the ischemic event
A) 1st, 2nd and 3rd answers are correct
B) 1st and 3rd answers are correct
C) 2nd and 4th answers are correct
D) only 4th answer is correct
E) all of the answers are correct

A

E) all of the answers are correct

In cerebral ischemia usually there is no abnormality on cranial CT scan in the first hours of the ischemic event. After 6-8 hours hyperacute ischemic signs could be seen. After 1-2 days clear hypodensity develops in the infracted territory. The hypodensity is mild and has blurred margin in the beginning, however, later it becomes demarcated. Contrast enhancement occurs in the subacute stage, and usually begins at the end of the first week of stroke. Contrast enhancement reaches its maximum at week 2 and 3, and fades over the following weeks. Contrast enhancement usually shows a gyriform pattern on the surface of the brain, but may also occur in the deep parenchyma. It is likely due to a combination of blood brain barrier breakdown, neovascularisation and impaired autoregulation. This phenomenon was previously referred to as ‘luxury perfusion’. Two-three weeks after ischemic stroke, the cortex may show near-normal density (isodense) on native CT, it is the so-called fogging phenomenon. Fogging phenomenon is explained by macrophage invasion, proliferation of capillaries, and sometimes extravasation of blood cells through damaged vessel walls. If in doubt, the administration of IV contrast will demarcate the region of infarction at this stage. Later (week 5-6) the residual swelling passes, and gliosis develops resulting in a region of low density with negative mass effect.

172
Q
Possible neurological complication of cardiac surgery:
1)  	myelon lesion
2)  	epileptic seizure
3)  	extrapyramidal symptoms
4)  	cortical blindness
A)  	1st, 2nd and 3rd answers are correct
B)  	1st tnd 3rd answers are correct
C)  	2nd and 4th answers are correct
D)  	only 4th answer is correct
E)  	all of the answers are correct
A

E) all of the answers are correct

173
Q

In which of the following case(s) should carotid duplex ultrasound examination be ordered?
1) after TIA
2) in case of cerebral haemorrhage to predict the prognosis
3) if auscultation reveals murmur over the carotid arteries
4) on the 7. day after subarachnoid bleeding to check for vasospasm
A) 1st, 2nd and 3rd answers are correct
B) 1st and 3rd answers are correct
C) 2nd and 4th answers are correct
D) only 4th answer is correct
E) all of the answers are correct

A

B) 1st and 3rd answers are correct

174
Q

Indication(s) for carotid endarteriectomy:
1) after TIA, if the stenosis of the internal carotid artery is larger then 70% measured by angiography
2) if the internal carotid artery is occluded on either side
3) in case of repeated or crescendo TIA on the side of the exulcerated plaque or floating thrombus
4) in case of acute stroke
A) 1st, 2nd and 3rd answers are correct
B) 1st and 3rd answers are correct
C) 2nd and 4th answers are correct
D) only 4th answer is correct
E) all of the answers are correct

A

B) 1st and 3rd answers are correct

175
Q
The average duration of migrain attacks is:
A)  	less, than 4 hours
B)  	0.5-1 day
C)  	4 days
D)  	one week	0.5-1 day
A

B) 0.5-1day

176
Q

What is lateral cordotomy?A) lesion in the ventral posterolateral nucleus of the thalamus
B) incision of the posterior funiculus in the spinal cord
C) a surgical procedure that injures the spinothalamic lateral tract in the spinal cord
D) electric stimulation of periaqueductal gray matter in the brainstem in order to alleviate severe pain

A

C) a surgical procedure that injures the spinothalamic lateral tract in the spinal cord

177
Q
What is the difference between neuralgic and radicular pain?
A)  	characteristics of the pain
B)  	intensity of the pain
C)  	the pain can not be well localized
D)  	duration of the pain
A

D) duration of the pain

neuralgic pain=seconds.
radicular pain=constant

178
Q
Bending down the head worsens the headache, if it is caused by:
A)  	glaucoma
B)  	maxillary sinusitis
C)  	tension type headache
D)  	cluster headache
A

B) maxillary sinusitis

179
Q
What is the prevalance of headache as a primary or associated symptom in brain tumors:
A)  	5–10%
B)  	20–30%
C)  	90–95%
D)  	60–70%
A

D) 60–70%

180
Q

Which of the following symptoms does not occur in migraine with aura?
A) hearing loss
B) speech disturbance
C) hemiparaesthesia
D) weakness of one of the the upper extremities

A

A) hearing loss

181
Q
The following aura signs may occur in migraine, except:
A)  	hemiparesis
B)  	hemiparaesthesia
C)  	numbness in both hands
D)  	aphasia
A

C) numbness in both hands

182
Q

Characteristics for pseudotumor cerebri:
A) It is rare during pregnancy
B) It is rarely associated with endocrinopathy or treatment with medication
C) Vventricules and subarachnoid space are wider than normal
D) Visual accuity can be severely affected
E) Headache is rare

A

D) Visual accuity can be severely affected

183
Q

Acoustic-neurinoma can cause:1) trismus
2) ear pain
3) trigeminal-neuralgia on the contralateral side
4) trigeminal-neuralgia on the ipsilateral side
A) 1st, 2nd and 3rd answers are correct
B) 1st and 3rd answers are correct
C) 2nd and 4th answers are correct
D) only 4th answer is correct
E) all of the answers are correct

A

D) only 4th answer is correct

184
Q

Which of the following headaches can predict/anticipate rupture of cerebral aneurysm:
1) migraine with aura
2) tension type of headache
3) short-lasting, extremely intense „predictive, anticipating” headache episodes (sentinel headache)
4) cluster headache
A) 1st, 2nd and 3rd answers are correct
B) 1st and 3rd answers are correct
C) 2nd and 4th answers are correct
D) only 4th answer is correct
E) all of the answers are correct

A

B) 1st and 3rd answers are correct

185
Q
Which of the following may cause painful polyneuropathy?
1)  	Lead intoxication
2)  	Diabetes mellitus
3)  	Chronic alcoholism
4)  	Multiple Sclerosis
A)  	1st, 2nd and 3rd answers are correct
B)  	1st and 3rd answers are correct
C)  	2nd and 4th answers are correct
D)  	only 4th answer is correct
E)  	all of the answers are correct
A

A) 1st, 2nd and 3rd answers are correct

186
Q
Which of the following may cause prosopalgia?
1)  	Glaucoma
2)  	Temporomandibular dyskinesia
3)  	Somatoform disorder
4)  	Allergic rhinitis
A)  	1st, 2nd and 3rd answers are correct
B)  	1st and 3rd answers are correct
C)  	2nd and 4th answers are correct
D)  	only 4th answer is correct
E)  	all of the answers are correct
A

B) 1st and 3rd answers are correct

187
Q
Radicular pain can be caused by:
1)  	extramedullary tumor
2)  	herpes Zoster
3)  	disc herniation
4)  	intramedullary tumor
A)  	1st, 2nd and 3rd answers are correct
B)  	1st and 3rd answers are correct
C)  	2nd and 4th answers are correct
D)  	only 4th answer is correct
E)  	all of the answers are correct
A

A) 1st, 2nd and 3rd answers are correct

188
Q
Which drug could cause toxic Parkinsonian syndrome?
A)  	anilin
B)  	zinc
C)  	MPTP (heroin derivate)
D)  	bensol
A

C) MPTP (heroin derivate)

189
Q

The relative amount of acetyl-choline in Parkinson’s disease is:
A) constant
B) elevated
C) decreased
D) there is no acetyl-choline in the striatum

A

B) elevated

compensation for the lack of dopamine.

190
Q

The focal dystonias (e.g.: blepharospasm):
A) typical in adulthood
B) typical in childhood
C) there are a lot of patients in both of these groups
D) inherited with the X-chromosome

A

A) typical in adulthood

191
Q
May be the symptom of Huntington’s chorea:
1)  	chorea
2)  	dementia
3)  	agression
4)  	muscle hypotonia
A)  	1st, 2nd and 3rd answers are correct
B)  	1st and 3rd answers are correct
C)  	2nd and 4th answers are correct
D)  	only 4th answer is correct
E)  	all of the answers are correct
A

E) all of the answers are correct

192
Q
May not cause parkinsonism as a side effect:
1)  	haloperidol
2)  	chlorpromazin
3)  	levomepromazin
4)  	clozapin
A)  	1st, 2nd and 3rd answers are correct
B)  	1st and 3rd answers are correct
C)  	2nd and 4th answers are correct
D)  	only 4th answer is correct
E)  	all of the answers are correct
A

D) only 4th answer is correct

193
Q

Ballism:
1) may be a symptom of Parkinson’s disease
2) hypertonic hyperkinesis
3) lesion of the fronto-ponto-cerebellar tract is etiological
4) caused by the lesion of subthalamic nucleus
A) 1st, 2nd and 3rd answers are correct
B) 1st and 3rd answers are correct
C) 2nd and 4th answers are correct
D) only 4th answer is correct
E) all of the answers are correct

A

D) only 4th answer is correct

194
Q

Acute stage of viral meningitis is not associated with:
A) common negative clinical findings of cranial CT
B) negative findings on EEG exam
C) cloudy liquor, elevated cell and protein count
D) elevated lymphocyte count in the CSF

A

C) cloudy liquor, elevated cell and protein count

CSF is clear or colorless.

195
Q
Initial stage of optic neuritis is characterized by central scotoma and:
A)  	papillary hyperaemia
B)  	papillary oedema
C)  	slight blur in vitreous humor
D)  	all of the above
E)  	none of the above
A

E) none of the above

196
Q

Characteristic symptoms of Guillain–Barre syndrome are:
1) flaccid paresis, absent deep reflexes
2) sensory deficit does not occur
3) cell-protein dissociation is commonly observed in the CSF
4) survival extends usually for a few months, maximum two years
A) answer 1, 2 and 3 correct
B) answer 1 and 3 correct
C) answer 2 and 4 correct
D) only answer 4 correct
E) all 4 answers correct

A

B) answer 1 and 3 correct

197
Q
Common clinical signs of multiple sclerosis are:
1)  	paraesthesia
2)  	optic neuritis
3)  	paresis
4)  	ataxia
A)  	answers 1, 2 and 3 are correct
B)  	answers 1 and 3 are correct
C)  	answers 2 and 4 are correct
D)  	only answer 4 is correct
E)  	all 4 answers are correct
A

E) all 4 answers are correct

198
Q
Spasticity in multiple sclerosis may be reduced using the following medications:
1)  	dantrolen
2)  	tizanidine
3)  	botulinumtoxin
4)  	baclofen
A)  	answer 1, 2 and 3 correct
B)  	answer 1 and 3 correct
C)  	answer 2 and 4 correct
D)  	only answer 4 correct
E)  	all 4 answers correct
A

C) answer 2 and 4 correct

199
Q

In multiple sclerosis, introduction of immunomodulatory treatment should be indicated:
1) in relapsing-remitting clinical form, during the active phase
2) in the primary, progressive form
3) in the early stage, until the patient retains mobility
4) in a secondary, progressive form, over age 50
A) answer 1, 2 and 3 correct
B) answer 1 and 3 correct
C) answer 2 and 4 correct
D) only answer 4 correct
E) all 4 answers correct

A

B) answer 1 and 3 correct

200
Q

The brain contusion almost always involves a loss of consciousness, and no abnormalities are visible on the CT scan and the CSF is often bloody.
A) true
B) false

A

B) false

Changes seen on CT.

201
Q

Multiple sclerosis is an autoimmune disease, in which T-cell mediated immune response has an important role.
A) true
B) false

A

A) true

EXPLANATIONThe exact pathomechanism of multiple sclerosis is unknown. According to one of the most widely accepted hypothesis, it’s a CD4 + T cell-mediated autoimmune disease in the central nervous system. The CD4 + T (Th1)-lymphocytes induce proinflammatory cytokines, leading to demyelization

202
Q

Which symptom is not characteristic for Myasthenia gravis?
A) diplopia
B) ptosis
C) dysarthria
D) aphasia
E) muscle weakness worsening with physical exercise

A

D) aphasia

203
Q
Frequent symptom in ALS:
A)  	dementia
B)  	agnosia
C)  	problems with urination
D)  	all
E)  	none
A

E) none

204
Q
Which disease is associated more frequently with polymyositis?
A)  	adrenoleukodystrophy
B)  	Lyme disease
C)  	SLE
D)  	viral hepatitis
A

C) SLE

205
Q
Which autoimmune disease may be associated with polyneuropathy?
1)  	Systemic Lupus Erythematosus 
2)  	rheumatoid arthritis
3)  	Wegener's-granulomatosis
4)  	Sjögren-syndrome
A)  	1st, 2nd and 3rd answers are correct
B)  	1st and 3rd answers are correct
C)  	2nd and 4th answers are correct
D)  	only 4th answer is correct
E)  	all of the answers are correct
A

E) all of the answers are correct

Alll are autoimmune diseases…

206
Q
Therapy for acute Guillain-Barré syndrome:
1)  	steroid
2)  	vitamins (B1 vitamin)
3)  	azathioprine
4)  	plasma exchange therapy
A)  	1st, 2nd and 3rd answers are correct
B)  	1st and 3rd answers are correct
C)  	2nd and 4th answers are correct
D)  	only 4th answer is correct
E)  	all of the answers are correct
A

C) 2nd and 4th answers are correct

207
Q

The differential diagnosis of amyotrophic lateralsclerosis (ALS) includes the following disease(s):
1) multiple sclerosis
2) multifocal motoric neuropathy
3) compression of the cervical spinal cord
4) cervical spondylosis
5) funicular myelosis
A) 1st, 2nd and 3rd answers are correct
B) 1st, 3rd and 4th answers are correct
C) 2nd, 3rd and 4th answers are correct
D) 2nd, 3rd and 5th answers are correct
E) 2nd, and 4th answers are correct
F) 3rd, 4th and 5th answers are correct

A

C) 2nd, 3rd and 4th answers are correct

ALS –> lower+uppper motor neurons. (no sensory involvement)
MS –> sensory
Multifocal motor neuropathy - only motor symptoms.